21 22Set2Test19HistoryfullsyllabusKEY

You might also like

Download as pdf or txt
Download as pdf or txt
You are on page 1of 81

SET 2 - TEST 19

HISTORY FULL SYLLABUS - EXPLANATION

Q.1) In the context of Indian history during 18th century, consider the following
statements:

1. Ahmad Shah Abdali defeated the Mughal army led by Shah Alam II in the historic third
battle of Panipat in 1761.

2. Farrukhsiyar was the first Mughal ruler to be killed by the treachery of nobles.

3. The Jagirdari crisis was one of the main reasons for the decline of Mughal empire in
India.

Which of the above given statements is/are correct?

(a) 1 and 2 only

(b) 2 and 3 only

(c) 3 only

(d) 1 and 3 only

EXPLANATION

The Third Battle of Panipat took place on 14 January 1761, at Panipat, about 60 miles north of
Delhi between a northern expeditionary force of the Maratha Empire and a coalition of the
King of Afghanistan, Ahmad Shah Durrani with two Indian Muslim allies—the Rohilla Afghans
of the Doab, and Shuja-ud-Daula, the Nawaz of Oudh. Martha's army was defeated by Ahmad
Shah Abdali in the 3rd Battle of Panipat (not in second battle of Panipat). So, statement 1 is not
correct.

Jahandar Shah was the first Mughal ruler who was killed by the Sayyid brothers- Abdullah Khan
and Hussain Ali. So, statement 2 is not correct.

The jagirdari system was a system that allotted jagirs to jagirdars or landlords in return for the
services rendered by them to the Mughal Empire. However, this led to bitter struggles among the
jagirdars for better swathes of land. Although this problem had started occurring during the Akbar
period, it became more serious during the period of the Later Mughals. In order to placate their
nobility and safeguard their power, the king gave away crown lands. This resulted in a major dip in
the flow of revenue to the royal treasury and inevitably led to a decline in the power of the Mughal
Emperor. Thus jagirdari crisis was one of the main reasons for the decline of the Mughal empire in
India. So, statement 3 is correct.
Q.2) In the context of 17th and 18th-century Indian history, consider the following
events:

1. Formation of independent states of Hyderabad, Bengal, and Awadh.

2. Invasion by Nadir Shah in India

3. Third Carnatic war

Arrange the above events in the correct chronological order of their occurrence:

(a) 1-2-3

(b) 2-1-3

(c) 1-3-2

(d) 3-1-2

EXPLANATION

After the death of Aurangazeb in 1707, the Mughal empire weakened, and the governor of
Hyderabad, Asaf Jah I (titled Nizam-ul-Mulk by the Mughal emperor), declared independence in
1724. His line ruled Hyderabad until 1948 and was called the Nizams. While Asaf Jah I ruled
from Aurangabad, his son and successor Nizam Ali Khan shifted the capital to Hyderabad, and a
glorious period started.

It was in the year 1739, Nadir Shar, the Shah of Iran and the founder of the Afsharid dynasty,
invaded India and brought about a boisterous and damaging mark on the history of Mughal
India.

The Third Carnatic war was fought between 1757 – 1763 between France and England.

The correct chronological order of their occurrence will be the Formation of independent states of
Hyderabad, Bengal, and Awadh - Invasion by Nadir Shah in India - Third Carnatic war. So,
option (a) is correct.

Q.3) The title of ‘Raja’ to Rammohan Rai was given by?

(a) Dwarka Nath Tagore

(b) William Bentick

(c) Akbar II

(d) Bahadur Shah II


EXPLANATION

The title of Raja to Rammohan Rai was given by Akbar II. Rammohan Rai was the founder of
the Brahmo Samaj, one of the first Indian socio-religious reform movements. He played a
major role in abolishing the role of Sati. Raja Rammohan Roy was a great scholar and an
independent thinker. He advocated the study of English, Science, Western Medicine, and
Technology. So, option (c) is correct.

ADDITIONAL INFORMATION

Rammohan Rai

 Raja Ram Mohan Roy was against idol worship and orthodox Hindu rituals.

 He worked as a moneylender in Calcutta, and from 1809 to 1814, he served in the


Revenue Department of the East India Company.

 In 1814, Raja Ram Mohan Roy formed Atmiya Sabha. Atmiya Sabha tried to initiate
social and religious reforms in society. Raja Ram Mohan Roy campaigned for rights for
women, including the right for widows to remarry, and the right for women to hold
property. He actively opposed the Sati system and the practice of polygamy.

 He also supported education, particularly the education of women. He believed that


English-language education was superior to the traditional Indian education system,
and he opposed the use of government funds to support schools teaching Sanskrit. In
1822, he founded a school based on English education.

 In 1828, Raja Ram Mohan Roy founded the 'Brahma Samaj'. Through 'Brahma Samaj,
he wanted to expose the religious hypocrisies and check the growing influence of
Christianity on Hindu society. Raja Ram Mohan Roy's efforts bore fruit when in 1929,
the Sati system was abolished.

 In 1831 Ram Mohan Roy traveled to the United Kingdom as an ambassador of the
Mughal emperor to plead for his pension and allowances. Raja Ram Mohan Roy passed
away on September 27, 1833, at Stapleton near Bristol due to meningitis.

Q.4) With reference to expansion of British rule in India, consider the following
statements:

1. The policy of Subsidiary alliance used by Wellesley was an extension of the policy of
ring fence followed by Warren Hastings.

2. Lord Dalhousie is considered as the originator of the doctrine of Lapse which was used
to annex Indian states.

3. The state of Awadh was annexed on the pretext of maladministration in 1856.


Which of the above given statements is/are correct?

(a) 1 only

(b) 2 and 3 only

(c) 1 and 3 only

(d) 1,2 and 3

EXPLANATION

The policy of the Subsidiary alliance was used by Wellesley which was an extension of the ring-
fence policy used by Warren Hastings to make the Indian States dependent on the British
Government.

Ring fence Policy: It was established to keep the states safe from external forces. The states have
to maintain the forces at their own expense.

Subsidiary alliance: This was the expanded form of the Ring Fence Policy. Under this system,
 Indian Rulers had to keep British troops in their territory.
 The ruler had to pay a subsidy for their maintenance.
 The ruler had to post one British resident in his court.
 They had to consult with the British before negotiating with any other European or, with
any other Indian ruler.
In return, the British would defend the ruler from external invasions, attacks of their enemies.
So, statement 1 is correct.

Lord Dalhousie is not considered as the originator of the policy of Doctrine of Lapse. It was
originally used by the British Court of Directors in 1834 to annex the princely states like Mandvi,
Kolaba, and Surat. So, statement 2 is not correct.

Dalhousie was keen on annexing the kingdom of Awadh. But the task presented certain
difficulties. For one, the Nawabs of Awadh had been British allies since the Battle of Buxar.
Moreover, they had been most obedient to the British over the years. The Nawab of Awadh had
many heirs and could not, therefore, be covered by the Doctrine of Lapse. Some other pretext had
to be found to deprive him of his dominions. Finally, Lord Dalhousie hit upon the idea of
alleviating the plight of the people of Awadh. Nawab Wajid Ali Shah was accused of having
misgoverned his state and of refusing to introduce reforms. His state was therefore
annexed in the pretext of maladministration in 1856. So, statement 3 is correct.

Q.5) The ‘Flood commission recommendations’ during the British rule was related to?

(a) Tebhaga movement of 1946

(b) Bardoli satyagraha


(c) Eka movement

(d) Telangana movement

EXPLANATION

Flood Commission recommendations are related to the Teghaga movement of 1946.

 The Tebhaga movement was a campaign initiated in Bengal by the Kisan Sabha in 1946–
47. At that time sharecropping peasants or Barghadars (essentially, tenants) had to give
half of their harvest to the owners of the land.

 The Tebhaga movement demanded to reduce the share given to landlords to one-
third. The Barghadars were encouraged by the fact that the Bengal Land Revenue
Commission, popularly known as the Floud Commission had already made this
recommendation in its report to the government.

 In 1946, the sharecroppers of Bengal began to assert that they would no longer pay a half
share of their crop to the Jotedars but only one-third. So, option (a) is correct.

Q.6) Consider the following statements with respect to Forward policy of Auckland:

1. The policy was mainly intended to protect the British empire from probable Russian
attack either through a treaty or annexation.

2. The policy led to the signing of a tripartite treaty between British, Sikhs and Shah
Shuja in 1838.

Which of the above given statements is/are correct?

(a) 1 only

(b) 2 only

(c) Both 1 and 2

(d) Neither 1 nor 2

EXPLANATION

Auckland who came to India as the governor-general in 1836, advocated a forward policy. This
implied that the Company government in India itself had to take initiatives to protect the
boundary of British India from a probable Russian attack. This objective was to be achieved
either through a treaty with the neighboring countries or by annexing them completely. So,
statement 1 is correct.

The forward policy led to a tripartite treaty between the British, Sikhs, and Shah Shuja in 1838.
The treaty provided that :
 Shah Shuja is enthroned with the armed help of the Sikhs.
 Shah Shuja to conduct foreign affairs with the advice of the Sikhs and the British;
 Shah Shuja give up his sovereign rights over Amirs of Sindh in return for a large sum of
money;

 Shah Shuja recognize the Sikh ruler, Maharaja Ranjit Singh’s claims over the Afghan
territories on the right bank of the River Indus.

So, statement 2 is correct.

Q.7) With reference to various acts passed by British before the revolt of 1857,
consider the following statements:

1. The Religious disabilities act of 1850 prevented the Indian sepoys from following their
religious rituals and customs while serving the British empire.

2. The General Service Enlistment act passed by Lord Canning mandated the sepoys to
give an undertaking to serve anywhere their services were required by the government.

3. The Lex Loci act provided the right to inherit ancestral property to Hindu converts to
Christianity.

Which of the above given statements is/are correct?

(a) 1 and 2 only

(b) 2 and 3 only

(c) 1 and 3 only

(d) 1,2 and 3

EXPLANATION

The Religious Disabilities Act was introduced in 1850. This law was passed in British India
under East India Company rule that abolished all laws affecting the rights of persons
converting to another religion or caste. It changed the Hindu Law of Property. It enabled a
convert from Hinduism to another religion to inherit the property of his father.It did not
prevented the Indian sepoys from following their religious rituals and customs while serving
the British empire So, statement 1 is not correct.

In 1856, Lord Canning’s government passed the General Service Enlistment Act which decreed
that all future recruits to the Bengal Army would have to give an undertaking to serve
anywhere their services might be required by the government. This caused resentment. So,
statement 2 is correct.
Lex Loci Act, which gave the Christian converts the right to inherit their ancestral properties.
The act, the Hindus widely believed, would open floodgates to Christian conversion. So,
statement 3 is correct.

Q.8) Which of the following were the reasons for the failure of the revolt of 1857?

1. Lack of central leadership.

2. Lack of Hindu-Muslim unity

3. No participation from Zamindars and landlords

4. Lack of clear understanding of colonial rule

Select the correct answer using the code given below:

(a) 1,2 and 3 only

(b) 1,3 and 4 only

(c) 2 and 4 only

(d) 1 and 4 only

EXPLANATION

The reasons for the failure of the revolt of 1857

1. All India participation is absent.

2. All classes did not join. Zamindars, Money lenders, and taluqdars did not join the revolt

3. Unorganized and Poorly organized. There is no Central leadership.

4. Poor arms and Equipment. The Indian soldiers were fighting with swords and spears and
very few guns and muskets.

During the revolt, there is complete coordination between the Hindus and Muslims at all levels.
All rebels acknowledged Bahadur Shah Zafar as the emperor and the first impulse of the Hindu
sepoys at Meerut was to March to Delhi.

So, option (b) is correct.

Q.9) With reference to various phases of British colonialism in India, consider the
following statements:

1. In the mercantile phase, political power was used to dictate terms of trade in favour of
English traders to monopolise the trading in Indian goods.

2. The industrial phase was based on the policy of one way free trade.
3. In the financial and the last phase, the capital of Britain was invested in the colonies
which increased its public debt.

Which of the above given statements is/are correct?

(a) 1 and 2 only

(b) 2 and 3 only

(c) 1 and 3 only

(d) 1,2 and 3

EXPLANATION

Mercantile Phase was from 1757 - 1813- This phase was marked by direct plunder. The East
India Company used it monopoly of trade which functioned through ‘investments’ of Indian
revenues to buy Indian products at low rates. These goods were then exported to Europe and
England. So in essence, the East India Company bought Indian products from the revenues they
collected mainly from Bengal and then exported them. Taking advantage of the political power the
British dictated the prices of the goods that they needed to export. So, statement 1 is correct.

The Industrial phase was based on One side Free Trade Policy: England wanted to sell the goods
made in European factories in the Indian market. But this was not feasible as they were of poor
quality compared to Indian products/things. So Indian industries have been deliberately
destroyed. During this regime, a one-sided policy for free trade was adopted, in which heavy
duties were imposed on the import of Indian cotton clothes in England, whereas there were no
charges/duties imposed on the goods imported from England. So, statement 2 is correct.

The third phase is seen to have begun from the 1860s, when British India became part
of the ever-expanding British Empire, to be placed directly under the control and sovereignty of
the British crown. This period was one of ‘finance imperialism when some British capital was
invested in the colony. This capital was organized through a closed network of British banks,
export-import firms, and managing agencies. So, statement 3 is correct.

Q.10) Consider the following statements:

1. He was associated with Bethune School in Calcutta to promote education for girls.

2. He was a pioneer in introducing western thoughts in the Sanskrit college which he also
made open for non brahmins.

3. He started a movement in support of widow remarriage which resulted in a legislation


widow remarriage act.

Which of the following social reformers is being described above?

(a) Ishwar chandra Vidyasagar


(b) BalShastri Jambhekar

(c) Jyotiba Phule

(d) Narayan Malhar Joshi

EXPLANATION

The great Scholar and reformed Vidyasagar’s ideas were a happy blend of Indian and Western
thought.

 In 1850, he became the principal of Sanskrit College.

 He was determined to break the priestly monopoly of scriptural knowledge, and for
this, he opened the Sanskrit College to non-brahmins.
 He introduced Western thought in Sanskrit College to break the self-imposed isolation of
Sanskritic learning. As an academician, he evolved a new methodology to teach Sanskrit.
He also devised a new Bengali primer and evolved a new prose style.

 Vidyasagar started a movement in support of widow remarriage which resulted in


legislation of widow remarriage. He was also a crusader against child marriage and
polygamy. He did much for the cause of women’s education.

 The Bethune School, founded in Calcutta, was the result of the powerful movement
for women’s education, that arose in the 1840s and 1850s. Ishwar Chandra
Vidyasagar was the secretary of Bethune School, where he was the pioneer of higher
education for women in India

So, option (a) is correct.

Q.11) Consider the following pairs:

Socio-religious Founded by initiative Reform

1. Satya Sodhak Samaj - Jyotiba Phule

2. Social service league - N.M Joshi

3. Seva Sadan - Gopal Krishna Gokhale

4. Prarthna Samaj - Keshub Chandra Sen

Which of the above pairs is/are correctly matched?

(a) 1 and 3 only

(b) 1 and 2 only

(c) 3 and 4 only

(d) 2 and 4 only


EXPLANATION

1. Satya Sodhak Samaj – It was founded by Jyotiba Phule in 1873, with the leadership of the
samaj coming from the backward classes, males, telis, Kunis, saris, and dangers. The main
aim of this movement were social service, spread of education among women’s and lower caste
people.

2. Social service league – Narayanan Malhar Joshi founded the social service league with an aim
to secure for the masses better and reasonable conditions of life and work.

3. Seva Sadan – It was founded by a social reformer Behramji M. Malabar in 1908 along with his
friend Diwan Dataram Gidumal. He vigorously spoke about child marriage and for widow
remarriage among Hindus.

4. Prathna Samaj – While Atmaran Pandurang founded the Prathna Samaj, Keshab Chandra Sen
only helped.

So, option (b) is correct.

Q.12) Which of the following movements is/are related to the Temple entry movement
in India in the 19th and 20th century?

1. Aruvippuram movement

2. Vaikom satyagraha

3. Justice movement

4. Radhaswami movement

Select the correct answer using the code given below:

(a) 1 and 2 only

(b) 2,3 and 4 only

(c) 1,3 and 4 only

(d) 1,2 and 3 only

EXPLANATION

Aruvippuram movement and Vaikom satyagraha are related to the Temple entry movement.
1. Aruvippuram movement - Narayana Guru, himself from the Ezhava caste, took a stone
from the Neyyar River and installed it as a Sivalinga at Aruvippuram on Sivaratri in 1888.
It was intended to show that the consecration of an idol was not the monopoly of the
higher castes. With this, he began a revolution that soon led to the removal of much
discrimination in Kerala’s society. The movement (Aruvippuram movement) drew the
famous poet Kumaran Asan as a disciple of Narayana Guru.
2. Vaikom satyagraha - In 1924, the Vaikom Satyagraha led by K.P. Kesava was launched in
Kerala demanding the throwing open of Hindu temples and roads to the untouchables.
The satyagraha was reinforced by jathas from Punjab and Madurai. Gandhi undertook a
tour of Kerala in support of the movement.

Justice Movement and Radhaswami movement is not related to the Temple entry movement. So,
option (a) is correct.

ADDITIONAL INFORMATION

RADHASWAMI MOVEMENT

 Tulsi Ram a banker from Agra, also known as Shiv Dayal Saheb, founded this
movement in 1861.

 The Radhaswamis believe in one supreme being, Supremacy of the guru, a company of
pious people (Satsang), and simple social life.

 Spiritual attainment, they believe, does not call for renunciation of the worldly life.

 They consider all religions to be true. While the sect has no belief in temples, shrines,
and sacred places, it considers as necessary duties, works of faith and chanty, service,
and prayer.

JUSTICE MOVEMENT

 This movement in Madras Presidency was started by C.N. Mudaliar, T.M. Nair, and P.
Tyagaraja to secure jobs and representation for the non-brahmins in the legislature.

Q.13) Which of the following statements is/are correct regarding Singh Sabha
movement:

1. The movement established khalsa schools to impart modern western education to the
sikhs.

2. It aimed to counter the proselytising activities of Christian missionaries and the Hindu
revivalists.

3. The Akali movement was an offshoot of the Singh sabha movement.

Which of the above given statements is/are correct?

(a) 1 only

(b) 2 and 3 only

(c) 1 and 3 only

(d) 1,2 and 3


EXPLANATION

One of the main objectives of the Singh Sabha movement is to make available modern western
education to the Sikhs. For this, a network of Khalsa schools was established by the Sabha
throughout Punjab. So, statement 1 is correct.

Another aim of the Singh Sabha movement is to counter the proselytizing activities of Christian
missionaries as well as Brahmo Samajists, Arya Samajists, and Muslim Maulvis. So, statement 2
is correct

The Akali movement also popularly known as the Gurudwara Reform Movement was an offshoot of
the Singh sabha movement. It aimed at liberating the Sikh gurudwaras from the control of corrupt
Udasi mahants (the post had become hereditary). These mahants were a loyalist and reactionary lot,
enjoying government patronage. The Akali Movement was a regional movement but not a communal
one. The Akali leaders played a notable role in the national liberation struggle though some
dissenting voices were heard occasionally. So, statement 3 is correct.

Q.14) Consider the following statements regarding the regulating act of 1773:

1. The act was the first parliamentary intervention in the affairs of British East India
company in India.

2. It created a Board of Control whose main responsibility was to supervise the affairs of
the Court of directors.

3. The Governors of Bombay and Madras were made subordinate to the Governor General
in council in matters related to war, revenue and diplomacy.

Which of the above given statements is/are correct?

(a) 1 and 2 only

(b) 1 only

(c) 1 and 3 only

(d) 2 and 3 only

EXPLANATION

The first important parliamentary act regarding the Company’s affairs was the Regulating Act of
1773. This Act made changes in the constitution of the Court of Directors of the Company and
subjected their actions to the supervision of the British government. So, statement 1 is
correct.

Board of Control was created by the Pits India Act 1784 (not by regulating act of 1773) to guide
and control the work of the court of directors and the Government of India. So, statement 2 is
not correct.

Pits Indian Act 1784 (not regulating act) clearly subordinated the Madras and Bombay
Presidencies to Bengal in all questions of war, diplomacy, and revenue. Regulating Act 1773
only made the Bombay and Madras were made subordinate to the Governor-General of Bengal,
it doesn’t clearly subordinate matters related to war, revenue, and diplomacy. So, statement 3
is not correct.

ADDITIONAL INFORMATION

Regulating Act 1773

 It designated the Governor of Bengal as the ‘Governor-General of Bengal’ and created an


Executive Council of four members to assist him. The first such GovernorGeneral was
Lord Warren Hastings.

 It provided for the establishment of a Supreme Court at Calcutta (1774) comprising one
chief justice and three other judges.

 It prohibited the servants of the Company from engaging in any private trade or
accepting presents or bribes from the ‘natives’.

 It strengthened the control of the British Government over the Company by requiring
the Court of Directors (governing body of the Company) to report on its revenue, civil,
and military affairs in India.

Q.15) In the context of British land revenue policy in India, consider the following
statements:

1. The permanent settlement system was introduced by Cornwallis to promote private


ownership of land by identifying zamindars as real land owners.

2. The Ryotwari settlement was applicable in the majority parts of India including Madras
and Bombay province.

3. The Mahalwari settlement was introduced by Thomas Munro and Alexander Reed.

Which of the above given statements is/are correct?

(a) 1 and 2 only

(b) 1 and 3 only

(c) 2 only

(d) 2 and 3 only


EXPLANATION

The Permanent settlement system was introduced by Lord Cornwallis which recognized the
Zamindars as the owners of the land and was given rights to collect the rent from the Peasants. It
was prevalent in British India which included the regions of Bengal, Bihar, Odisha, Banaras region
of Uttar Pradesh, and areas of North Karnataka. 1/11th of the revenue could be retained by
zamindars while 10/11th was to be submitted to the Company. It promoted private ownership of
land by identifying zamindars as real land owners.So, statement 1 is correct
The Ryotwari settlement was introduced by Thomas Munro and Alexander Reed. Read and Munro
felt that in the south there were no traditional zamindars. The settlement, they argued, had to be
made directly with the cultivators (ryots) who had tilled the land for generations. Their fields had to
be
carefully and separately surveyed before the revenue assessment was made. It was effective in the
area of British India which included regions of Madras, Bombay, Eastern Bengal, Assam, and
Koorg. So, statement 2 is correct
In the North-Western Provinces of the Bengal Presidency (most of this area is now in Uttar
Pradesh),
an Englishman called Holt Mackenzie devised the new system called Mahalwari settlement
which came into effect in 1822. He felt that the village was an important social institution in
north Indian society and needed to be preserved. Under his directions, collectors went from village
to village, inspecting the land, measuring the fields, and recording the customs and rights of
different groups. The estimated revenue of each plot within a village was added up to calculate the
revenue that each village (mahal) had to pay. This demand was to be revised periodically, not
permanently fixed. The charge of collecting the revenue and paying it to the Company was given to
the village headman, rather than the zamindar. So, statement 3 is not correct

Q.16) Consider the following statements regarding the evolution of civil service during
British India:
1. The charter act of 1833 ended the Company’s patronage by providing recruitment
through an open competition.

2. Satyendra Nath Tagore was the first Indian to qualify for the Indian civil service.

3. Fort William college was set up by Lord Cornwallis for the training of new recruits to the
civil services.

Which of the above given statements is/are correct?

(a) 1 only

(b) 2 only

(c) 2 and 3 only

(d) 1 and 2 only


EXPLANATION

The Charter Act 1853(not 1833) introduced an open competition system of selection and
recruitment of civil servants. The covenanted civil service was, thus, thrown open to the Indians
also. Accordingly, the Macaulay Committee (the Committee on the Indian Civil Service) was
appointed in 1854. So, statement 1 is not correct.

Satyendra Nath Tagore was the first Indian to join the Indian Civil Service. He was posted to the
Bombay ICS, where he served his entire career from 1864 to 1897. So, statement 2 is correct.

Fort William College was set up by Lord Wellesley for the training of recruits. In 1806 Wellesley's
college was disapproved by the Court of Directors and instead the East Indian College was set up.
So, statement 3 is not correct.

ADDITIONAL INFORMATION

INDIAN CIVIL SERVICE ACT 1861

 This Act reserved certain offices for covenanted civil servants but the examination was
held in England in English language, based on classical learning of Greek and Latin.
The maximum permissible age was gradually reduced from 23 (in 1859) to 22 (in 1860)
to 21 (in 1866) and to 19 (1878)

Q.17) The term ‘Responsible government’ used in the context of British administration
in India can be best described by?

(a) A government which is based on the principle of rule of law

(b) A government which is responsible to the needs of the poor people of India

(c) A government where the Governor was to act on the advice of ministers who were
responsible to the legislature.

(d) A government which allowed Indian participation in the bureaucracy

EXPLANATION

The term Responsible Government used in the context of British administration in India can
be best described by a Government where the Governor was to act on the advice of ministers
who were responsible to the legislature. Responsible government is a conception of a system of
government that embodies the principle of parliamentary accountability. So, option (c) is
correct.
Q.18) Consider the following statements with respect to All India trade union
congress(AITUC):

1. Lala Lajpat Rai was the founding member of the union formed in 1920 to support the
cause of the working class.

2. It later merged into the Congress Socialist Party.

3. It adopted a resolution of Swaraj in its second session held in Jharia in 1921.

Which of the above given statements is/are correct?

(a) 1 only

(b) 2 and 3 only

(c) 1 and 3 only

(d) 1,2 and 3

EXPLANATION

The All India Trade Union Congress (AITUC) is the oldest trade union federation in India, founded on
31 October 1920 in Bombay. Among its founders were the renowned nationalist leader Lala Lajpat
Rai, who became the first President of the AITUC, Joseph Baptista, N.M. Joshi (1879-1955) and
Diwan Chaman Lall. The formation of the AITUC led to it being the designated official representative
of the Indian workers at the International Labour Organisation (ILO) from 1921 onwards. So,
statement 1 is correct.

AITUC has been always remained separate and does not merged into Congress socialist party. So,
statement 2 is not correct.

At the second annual session of the Trade Union Congress in Jharia 1921, a Swaraj resolution was
passed which was earlier than that, INC has only passed the swaraj resolution in 1930. So,
statement 3 is correct.

Q.19) With regard to peasant movements in India, consider the following statements:

1. The peasant movements of the 19th century were more localised in their demands and
were not against the colonial rule of British.

2. The Kisan Sabha movement was mainly inspired by the home rule activists and the first
kisan sabha was formed in Uttar Pradesh.

3. Mappila Revolt was mainly directed against the exploitative policy of land revenue by
Britishers.

Which of the above given statements is/are correct?

(a) 1 and 2 only


(b) 2 and 3 only

(c) 1 and 3 only

(d) 1,2 and 3

EXPLANATION

The peasant movements of the 19th century were localized in their demands and were not against
the colonial rule of the British. They fought only against zamindars and the policies of British but
were not aimed at abolishing colonial rule.The peasants suffered from high rents, illegal levies,
arbitrary evictions and unpaid labour in Zamindari areas. The Government levied heavy land
revenue. For example the Indigo revolt is completely localised where the Peasants revolted against
the Planters who compelled the Peasants for forced cultivation of indigo in Bengal region. So,
statement 1 is correct.

Kisan Sabhas came up in the United Provinces mainly due to the efforts of the Home Rule
activists. The UP Kisan Sabha was formed in February 1918 under the leadership of Gauri
Shankar Mishra and Indra Narayan Dwivedi and was supported by Madan Mohan Malaviya. So,
statement 2 is correct.

Mappila revolt also called as Moplah rebellion is led by the Mappila Muslims of Kerala against the
exploitative land revenue policy by the Hindu landlords. This the revolt was completely against the
Hindu landlords and not against the British. So, statement 3 is not correct.

Q.20) In the context of British rule in India, which of the following events happened
earliest?

(a) A police commission was established under Andrew Frazer to review police
administration.

(b) Younghusband’s mission to Tibet

(c) Ilbert bill controversy

(d) Curzon- kitchener controversy

EXPLANATION

1. In 1902-03, a Police Commission was established for the Police reforms under Sir Andrew
Frazer and Lord Curzon. It recommended the appointment of Indians at the officer level in the
police. Indians could rise only to the ranks of Inspector of Police, the senior N.C.O. position.
However, they were not part of the Indian Imperial Police.
2. Lord Curzon sent the Young husband’s mission to Tibet to revisit the Russian infiltration to
Tibet and solve the boundary disputes between Tibet and Sikkim. It was a Military expedition
started in 1903 and lasted till 1904.
3. Ilbert Bill, in the history of India, was a controversial measure proposed in 1883 that sought to
allow senior Indian magistrates to preside over cases involving British subjects in India.
4. Lord Kitchener (1850-1916) arrived in Bombay to assume his post as Commander-in-Chief of
the Indian Army. In January 1903, Lord Kitchener began the use of a series of unofficial
communication channels with General Sir Edward Stedman (1842-1925), Military Secretary to
the India Office, and Lady Cranborne, later Lady Salisbury, who was a close friend of the Prime
Minister, Arthur Balfour.
 In February, Lord Kitchener made his first formal proposal to the Viceroy, Lord Curzon,
regarding the reduction of powers assigned to the position of the Military Member.
 Lord Kitchener objected to the organization of the command structure of the Indian
Army which had emanated from the Indian Councils Act of 1861. It allowed the
Commander-in-Chief to be appointed as needed to an ex officio position on the Viceroy's
Executive Council, while the officer in charge of the Military Department possessed a
permanent Council seat.
 Lord Curzon rejected Kitchener's approach and asked him to study the existing system
for a year.
 The controversy was between 1902 – 1905.
The earlier event held was the Ilbert Bill Controversy. So, option (c) is correct.

Q.21) Consider the following statements with regard to Pabna Agrarian leagues:

1. The leagues were formed by Bengal peasants to protest against the Act X of 1859 which
empowered the zamindars to collect enhanced rent.

2. The main source of their struggle was use of legal resistance and there was negligible
violence.

3. The protests led to the passing of the Bengal tenancy act in 1885 to recognise the rights
of tenant farmers.

Which of the above given statements is/are correct?

(a) 1 only

(b) 2 and 3 only

(c) 1 and 3 only

(d) 3 only

EXPLANATION

The Pabna agrarian revolt was led by the Peasants against the Bengal landlords who forcefully
collected the land revenue from the Peasants. The Act X of 1859 conferred occupancy rights on
tenants who held land for 12 years as well as paid their rents. But zamindars didn't want this to
happen. The peasants were often evicted from the land due to non-payment. The Peasants only
protested against the Zamindars not against the Act X of 1859. So, statement 1 is not correct.
The Pabna Agrarian leagues were carried by legal resistance and there was only a little violence.
Mostly the problem was solved by the Official persuasion. So, statement 2 is correct.
Bengal Tenancy Act 1885 was an enactment of the Bengal government defining the natural rights
and liabilities of zamindars and tenants in response to widespread peasant discontent (Pabna
Agrarian leagues) threatening the stability of the colonial system of governance. So, statement 3 is
correct.

Q.22) Consider the following pairs:

Tribal revolts Persons associated

1. Kandh uprising - Chakra Bisoi

2. Ahom revolt - Gomdhar Konwar

3. Pagal panthis - Karam shah

4. Bhil uprisings - Gaidiliu

Which of the above given pairs is/are correctly matched?

(a) 1,2 and 3 only

(b) 2,3 and 4 only

(c) 1 and 3 only

(d) 1,2 and 4 only


EXPLANATION

1. Kandh Uprising - The Kandhs retaliated under Chakra Bisoi against the British
efforts to put an end to the Kandh’s practice of human sacrifice (Mariah) first through
persuasion and later through force. The Kandhs fought with tangis—a sort of battle
axe—bows and arrows and even swords.

2. Ahom revolt - The British had pledged to withdraw after the First Burma War (1824-
26) from Assam. But, after the war, instead of withdrawing, the British attempted to
incorporate the Ahoms’ territories in the Company’s dominion. This sparked off a
rebellion in 1828 under the leadership of Gomdhar Konwar. Finally, the Company
decided to follow a conciliatory policy and handed over Upper Assam to Maharaja
Purandar Singh Narendra and part of the kingdom was restored to the Assamese
king.
3. Pagal Panthis - The Pagal Panthi, a semi-religious group mainly constituting the
Hajong and Garo tribes of Mymensingh district (earlier in Bengal), was founded by
Karam Shah.The tribal peasants organized themselves under Karam Shah's son
Tipu to fight the oppression of the Zamindars.
4. Bhil uprisings - It was Govind Guru, the social and religious leader of the Bhil
community for more than three decades, who was the pivotal force in uniting Bhils to
demand a separate entity in the British period. The movement led to the infamous
Mangarh massacre on November 17, 1913.

So, option (a) is correct.

Q.23) Which of the following Organisations merged to form the British Indian
Association in 1851?

1. The Zamindari Association

2. The Bengal British India society

3. East India Association

Select the correct answer using the code given below:

(a) 1 and 2 only

(b) 2 and 3 only

(c) 1 and 3 only

(d) 1,2 and 3


EXPLANATION

The Zamindari Association and the Bengal British Indian society were merged to form the
British Indian Association in 1851.

 British Indian Association, was founded on October 29, 1851, at Calcutta with Raja
Radha Kanta dev and Debendranath Tagore as its President and Secretary
respectively.
 Other members of the Association included Ramgopal Ghosh, peary chand Mitra,
and Krishnadas Pal. Its membership was kept exclusive to Indians.

 The object of the Association was 'to secure improvements in the local
administration of the country and the system of government laid down by
Parliament'.

 The Association gave leadership to remove the existing defects in the laws and civil
administration of the country and to promote the greater welfare of the Indians.

 The Association informed parliament that Indians were not benefited by their
connection with Great Britain 'to the extent they expected'. So, option (a) is
correct.

Q.24) Consider the following:

1. Mahadeo Govind Ranade

2. Badruddin Tyabji

3. Pherozshaha Mehta

4. K.T Telang

Who among the above leaders were the founding members of the Bombay Presidency
Association?

(a) 1,2 and 3 only

(b) 2 and 3 only

(c) 2,3 and 4 only

(d) 1 and 4 only

EXPLANATION

Along with Pherozeshah Mehta and Kashinath Trimbak Telang, Badruddin Tyabji was largely
responsible for forming the Bombay Presidency Association in 1885, a body that championed
Indian interests and hosted the first meeting of the Indian National Congress in Bombay at the end
of 1885. So, option (c) is correct
Q.25) In the context of British colonial rule, the Inland Emigration act of 1859 was
related to which of the following?

(a) Indigo farmers in Bengal and Bihar

(b) Peasants of the Deccan region

(c) Poligars of the south

(d) Plantation workers of Assam

EXPLANATION

For plantation workers in Assam, freedom meant the right to move freely in and out of the
confined space in which they were enclosed, and it meant retaining a link with the village from
which they had come.
Under the Inland Emigration Act of 1859, plantation workers were not permitted to leave the
tea gardens without permission, and in fact, they were rarely given such permission. When they
heard of the Non-Cooperation Movement, thousands of workers defied the authorities, left the
plantations, and headed home. They believed that Gandhi Raj was coming and everyone would be
given land in their own villages.
So, option (d) is correct.

Q.26) Consider the following statements regarding DadaBhai Naroji:

1. He was the President of Indian National congress for five times.

2. He led a British committee of INC which was established in 1899 to mobilise the British
public opinion to introduce reforms in India.

3 He was also one of the founding members of the United Patriotic association formed in
Varanasi.

Which of the above-given statements is/are correct?

(a) 1 and 2 only

(b) 2 only

(c) 1 and 3 only

(d) 2 and 3 only

EXPLANATION

Dadabhai Naroji was elected as President of INC three times in 1886, 1893, and 1906. So,
statement 1 is not correct.
The British committee was established in Britain by INC. Dadabhai Naoroji, when he was an MP in
London, attended this group's meetings, and was associated with their parliamentary pressure
group. In 1890, the committee began to produce India, a free monthly journal summarising Indian
news for the British press and politicians. India became a weekly subscribed journal, 1898-1921.
Its editors included Henry Cotton (1906-19) and Henry Polak (1919-20). Its purpose is to raise
awareness of Indian issues to the public in Britain. So, statement 2 is correct.

Sir Syed Ahmed Khan founded the Indian Patriotic Association with the support of some Hindu
and Muslim leaders to oppose the Congress and tried to dissuade the Muslims from joining the
congress. He also emphasized the unity between Hindus and Muslims. Dadabai Naroji was not the
founding member. So, option (b) is correct.

Q.27) Consider the following statements regarding Indian Council Act, 1892:

1. The act for the first time introduced an element of election to legislative councils.

2. The budget was allowed to be discussed but could not be voted upon by the legislative
members.

3. The majority in the council was retained by the officials or the executives.

Which of the above given statements is/are correct?

(a) 2 and 3 only

(b) 1 only

(c) 1 and 2 only

(d) 1,2 and 3

EXPLANATION

The Indian Councils Act of 1892 introduced the element of election for the first time. Some of the
members of Imperial Legislative Assemblies and Provincial Legislative Assembly could be indirectly
elected. So, statement 1 is correct.

It increased the functions of legislative councils and gave them the power of discussing the budget
and not to vote and addressing questions to the executive. So, statement 2 is correct.

It increased the number of additional (non-official) members in the Central and Provincial legislative
councils but maintained an official majority in them. So, statement 3 is correct.
Q.28) With reference to sessions of Indian National Congress, consider the following
statements:

1. First session of Indian National Congress was held at Calcutta.

2. Kadambini Ganguly became the first woman president of INC in 1890.

3. Congress accepted Swaraj as its goal in the 1906 session.

Which of the above given statements is/are correct?

(a) 1 and 2 only

(b) 3 only

(c) 1 only

(d) 2 and 3 only

EXPLANATION

The first session of the Indian National Congress was held in December 1885 in Bombay with
seventy-two delegates. More than just a political party, Congress was an assembly for politically-
minded individuals who were interested in reform. In its first twenty years, known as a 'moderate
phase', Congress was not interested in campaigning for independence or self-rule but for greater
political autonomy within the empire. So, statement 1 is not correct.

Annie Beasant was the first women President of INC who presided over the Calcutta session in
1917. Kadambini Ganguly,was the first Indian women president of INC. So, statement 2 is not
correct.

Congress in 1906 at Calcutta session under Dadabhai Naoroji accepted the Swaraj as the goal.
So, statement 3 is correct.

Q.29) In the context of British colonial rule, which of the following events happened
during the Viceroyship of Lord Chelmsford?

1. Lucknow pact between congress and Muslim league

2. Montagu’s August declaration

3. Death of Tilak

4. Chauri Chaura incident

Select the correct answer using the code given below:

(a) 1 and 2 only

(b) 2,3 and 4 only


(c) 1,2 and 3 only

(d) 1 and 4 only

EXPLANATION

The following events took place during the Viceroyship of Lord Chelmsford

1. Lucknow pact between Congress and Muslims - The Congress-League Scheme 1916, also
known as the Lucknow Pact, was a document jointly drafted by the Indian National
Congress and the All India Muslim League. In 1913, the League at its Lucknow session –
which Mohammad Ali Jinnah attended for the first time - decided to cooperate with the
Congress and join in on calls for ‘self-government’. The Congress, in the same year, held its
28th session at Bombay. At this session, which Jinnah attended as a delegate, the Congress
appreciated the League’s stance on self-government and laid the foundation for
collaboration between the League and the Congress.

2. Montagu's August Declaration - The Secretary of State for India Montague made a statement
on August 20, 1917, in the British House of Commons in what has come to be known as
the August Declaration of 1917. The statement said: “The government policy is of an
increasing participation of Indians in every branch of administration and gradual
development of self-governing institutions with a view to the progressive realization of
responsible government in India as an integral part of the British Empire.”

3. Death of Tilak - He died on August 1 1920due to the illness. Montagu was the Viceroy
when Tilak died.

4. Chauri Chaura incident - The Chauri Chaura incident occurred at Chauri Chaura in the
Gorakhpur district of the United Province, (modern Uttar Pradesh) in British India on 4
February 1922, when a large group of protesters, participating in the Non-cooperation
movement, clashed with police, who opened fire. Lord Reading was the Viceroy when this
incident was happened.

So, option (c) is correct.

Q.30) Among the committees related to education during colonial rule, which of the
following happened earliest?

(a) Hunter Education commission

(b) Saddler university commission

(c) Hartog committee

(d) Sergeant plan of Education


EXPLANATION

Hunter Education commission - Lord Ripon Governor-General of India appointed the first
Indian Education Commission on February 3, 1882, under the Chairmanship of Sir William
Hunter, a member of the Executive Council of Viceroy. So this Commission is popularly
known as Hunter Commission. The Government desired that “the commission should
specially bear in mind the great importance which the Government attaches to the subject of
primary education”. Though the development of primary education was one of the main
objects contemplated by the Woods Despatch, 1854, yet owing to the variety of
circumstances expected results could not be achieved in the field of primary education.

Saddlers University commission - This commission was appointed in 1917 to study and
report the problem of University education. The commission was appointed under the
chairmanship of M.E. Saddler who was the Vice-Chancellor of the University of Leeds. The
Saddler University Commission also included two Indian Members, namely Sir Ashutosh
Mukherjee and Zia-ud-din Ahmed. The Saddler University Commission reported that the
condition of secondary education needs to be improved in order to improve the standard of
university Education. The commission visited all the university centers and after 17 months,
submitted its report in 1919. It is a very long and significant report.
Hartog Committee - The Committee studied the various aspects of education and
submitted its report before the commission in 1929, It put forward comprehensive
recommendations concerning various facts of education in India. First, the Committee made
some general observations regarding the state of education in India. The committee observed
that there was considerable progress made in education by the time. In general, people
regarded education as a matter of national importance. Increasing enrolment in primary
school indicates that the sense of indifference to education was breaking down and social
and political consciousness among the people had also increased. The women, the Muslims,
and the backward classes had also awakened and there had been rapid progress in the
numbers.

Sergeant plan of Education - The government of British India formed a Committee of


Enquiry with 22 members. The report of the committee was submitted to the Central
Advisory Board of Education (CABE) in 1944. The Board accepted its recommended
enforcement. The scheme was known as the ‘Sargent Scheme of Education’ as it was
prepared by John Sargent. It is also known as ‘Report by the Central Advisory Board of
Education’ and also as the plan for post-war educational reconstruction in India. This
scheme has historical importance as it was the first attempt to develop a National System for
Education in India. The report of the Committee consisted of 12 different chapters covering
from pre-primary to university education. It was a full-fledged educational plan for the future
educational reconstruction in India. The report had diagnosed every problem critically and
had given definite and clear-cut solutions. It deals with almost all types of education for all
classes of people in India. This was the first report that presents a comprehensive picture of
education in our country at that period of time. The report is undoubtedly a valuable
educational document.

Thus,Hunter Education Commission was formed earlier. So, option (a) is correct.

Q.31) Consider the following statements regarding Kuka revolt:

1. The movement started as a religious purification movement in Punjab and later became
a political one.

2. The movement discouraged modernisation and western education among women.

3. It was founded by Bhagat Jawar Mal who was also known as Sian Saheb.

Which of the above given statements is/are correct?

(a) 1 only

(b) 1 and 3 only

(c) 1 and 2 only

(d) 2 and 3 only

EXPLANATION

After the British took Punjab, the movement transformed from a religious purification campaign
to a political one. Its basic tenets were the abolition of caste and similar discriminations among
Sikhs, discouraging the eating of meat and taking of alcohol and drugs, and encouraging women
to step out of seclusion. So, statement 1 is correct.

Kukas called for a boycott of educational institutions of British and laws established by them.
They were rigid in their clothing and wore only hand-spun white attire. The Kuka followers
actively propagated civil disobedience. The movement does not discourage women’s education.
So, statement 2 is not correct.

The Kuka Movement was founded in 1840 by Bhagat Jawahar Mai also called Sian Saheb in
western Punjab. So, statement 3 is correct.

Q.32) With reference to Women participation in the Indian freedom struggle, consider
the following statements:

1. The coming of Gandhiji to the national movement can be considered as a watershed


moment in the participation of women in Indian freedom struggle.
2. Sarla Devi Chaudhurani was the founding member of Arya Mahila Samaj

3. Stri Dharma was a monthly magazine published by Women’s Indian Association for the
upliftment of Women.

Which of the above given statements is/are not correct?

(a) 1 and 2 only

(b) 3 only

(c) 2 only

(d) None

EXPLANATION

Gandhiji accorded special role to women in Non cooperation Movement and Civil Disobedience
Movement, he glorified their strength despite accepting their biological and social limitations. Salt
Satyagraha was the first nationalist activity in which women participated in large numbers. The
socialist activist Kamaladevi Chattopadhyay had persuaded Gandhiji not to restrict the protests to
men alone. Kamaladevi was herself one of the numerous women who courted arrest by breaking the
salt or liquor laws. Thus coming of Gandhi to the national movement can be considered as a
watershed moment in the participation of women in the Indian freedom struggle. So, statement 1
is correct.

The Arya Mahila Samaj was started on November 30, 1882, by Pandita Ramabai with the aim of
empowering and educating each woman for leading a dignified life. The inaugural meeting was
attended by 150 women. On the first day, 16 women members were enrolled. We are in our 136th
year and the Arya Mahila Samaj has turned into a trust. Sarla Devi Chaudhurani founded the All
India Organisation for women’s called Bharat Street Mahamandal. Hence she was not the founding
member of Arya Mahila Samaj. So, statement 2 is not correct.

The journal Stri-Dharma a monthly magazine was published by the Women's Indian Association
from 1918 to 1936, endeavored to be the voice of the Indian women's rights movement. It addressed
political and social issues facing women in India as well as the achievements of women worldwide.
So, statement 3 is correct.

Q.33) In the context of Indian freedom struggle, arrange the following events in the
correct chronological order:

1. Partition of Bengal

2. Establishment of Indian society of oriental art

3. Establishment of National council of education

4. Passing of Indian Universities act


Select the correct answer using the code given below:

(a) 1-4-2-3

(b) 4-1-3-2

(c) 4-1-2-3

(d) 2-4-1-3

EXPLANATION

1. Partition of Bengal - 1905

2. Establishment of Indian society of oriental art – 1907

3. Establishment of National council of education – 1906

4. The passing of the Indian Universities act – 1904

The chronological order will be, Passing of Indian Universities act – Partition of Bengal -
Establishment of National council of education - Establishment of Indian society of oriental
art. So, option (b) is correct.

Q.34) With reference to Swadeshi and Boycott movement, consider the following
statements:

1. A formal proclamation of swadeshi movement was made in the 1905 congress session
presided over by Gopal Krishna Gokhale.

2. There was no muslim participation in the swadeshi movement.

3. Swadesh Bandhab samiti was formed under Ashwini Kumar Dutta to spread political
consciousness among the masses.

Which of the above given statements is/are correct?

(a) 1 only

(b) 2 and 3 only

(c) 1 and 3 only

(d) 3 only

EXPLANATION

The Congress session at Banaras in 1905 made a formal proclamation of the swadeshi movement
which was presided over by President Gopala Krishna Gokhale. The swadeshi movement was a
proclamation of reaction towards the Partition of Bengal. So, statement 1 is correct.

Most of the Upper and Middle-class Muslims didn’t participate in the Movement. They supported
the partition in the plea that it would give them Muslim majority East Bengal. However, some of
the Muslims who participated were Barrister Abdul Rasul, Liaqat Hussain, Guzanvi, Maulana Azad
(who joined one of the revolutionary terrorist groups). So, statement 2 is not correct.

Samitis such as the Swadesh Bandhab Samiti of Ashwini Kumar Dutta (in Barisal) emerged as a
very popular and powerful method of mass mobilization. These samitis generated political
consciousness among the masses through magic lantern lectures, swadeshi songs, physical and
moral training to their members, social work during famines and epidemics, organization of
schools, training in swadeshi crafts, and arbitration courts. So, statement 3 is correct.

Q.35) In the context of revolutionary activities, which of the following is not correctly
matched?

(a) Anushilan Samiti- Promotha Mitter

(b) Mitra Mela- V.D Savarkar

(c) Ramsoi Peasant force- Vasudev Balwant Phadke

(d) Anjuman-i- Mohisban-i- Watan- Bhagat Singh

EXPLANATION

Anjuman-i- Mohisban-i- Watan was organized by Anita Singh (Bhagat Singh’s Uncle) in Lahore with
its journal Bharat Mata. So, option (d) is correct.

ADDITIONAL INFORMATION

ANUSHILAN SAMITI

 It was formed by Promotha Mitter and including Jatinfranath Banerjee, Barindra


Kumar Ghosh.

 Their activities were limited to giving physical and moral training to the members and
remained insignificant till 1907 – 08.

MITRA MELA

 Savarkar and his brother organized Mitra Mela a secret society in 1899 whoch merged
with Abhinav Bharat in 1904.

 Soon Nasik, Poon, and Bombay emerged as centres of bomb manufacture, A.M.T.
Jackson, the Collector of Nasik, who was also a well-known indologist, was killed by
Anant Lakshma Kanhere, a member of Abhinav Bharat.
 At the trial, Savarkar as the soul, inspiration, and moving spirit of the conspiracy
extending over a number of years, was sentenced to transportation for life and forfeiture
of all his property.

Ramsoi Peasant force

 The first of the revolutionary activities in Maharashtra was the organization of the
Ramosi Peasant Force by Vasudev Balwant Phadke in 1879, which aimed to rid the
country of the British by instigating an armed revolt by disrupting Communication
lines.

 It hoped to raise funds for its activities through dacoities. It was suppressed
prematurely.

Q.36) The idea of Muslim league for the first time was floated by which of the following
personalities?

(a) Agha Khan

(b) Sir Syed Ahmed Khan

(c) Nawab Salimullah

(d) Muhammad Ali Jinnah

EXPLANATION

The idea of the Muslim league was first proposed by Nawab Salimullah of Dacca on December 30,
1905, as an anti–Congress front. So, option (c) is correct.

Q.37) Consider the following statements;

1. The Home rule leagues led by Tilak and Annie Besant played a decisive role in reuniting
Extremists into the fold of congress.

2. The Locknow pact was signed between Congress and Muslim league in 1916 with the
former conceding to the demand of separate electorate.

3. The Anglo -Indians , most muslims and non Brahmins from the south did not join the
Home rule leagues.

Which of the above given statements is/are correct?

(a) 1 and 2 only

(b) 2 and 3 only

(c) 1 and 3 only

(d) 1,2 and 3


EXPLANATION

The efforts of Tilak and Annie Besant towards the Moderate – Extremist reunion at Lucknow revived
the Congress as an effective instrument of Indian Nationalism. It helped in reuniting Extremists into
the fold of Congress. So, statement 1 is correct.

Lucknow pact between the Congress and the Muslim League was a significant development, which
paved a way for the presentation of common demands by them to the government. While the league
agreed to present joint constitutional demands with Congress the Congress accepted the Muslim
league position on demand for the separate electorates. So, statement 2 is correct.

Most of the Anglo -Indians, most Muslim’s and non-Brahmins from the south did not join the Home
rule leagues as they felt Home rule would mean the rule of the Hindu majority and that too mainly
by the high caste. So, statement 3 is correct.

Q.38) Which of the following British colonial acts gave Women the right to vote for the
first time?

(a) Morley Minto reforms, 1909

(b) The Government of India act, 1919

(c) The Government of India act, 1935

(d) Indian Council act, 1892

EXPLANATION

In line with the government policy contained in Montagu’s statement of August 1917, the government
announced further constitutional reforms in July 1918, known as Montagu-Chelmsford or Montford
Reforms. Based on these, the Government of India Act, 1919 was enacted. This was the first British
act that gave women the right to vote. So, option (b) is correct.

Q.39) With reference to Mahatma Gandhi campaign in South Africa, consider the
following statements:

1. He started a paper namely Indian Opinion to unite different sections of Indiana residing
in South Africa.

2. A Passive resistance association was formed to conduct a campaign against the


registration certificates required to be carried by Indians in South Africa.

3. Gandhi ji set up a Tolstoy farm to house the families of satyagrahis and to provide them
a source of livelihood.

Which of the above given statements is/are correct?


(a) 1 only

(b) 2 and 3 only

(c) 1 and 3 only

(d) 1,2 and 3

EXPLANATION

During the moderate phase (1894 – 1906) phase, Gandhi relied on sending petitions and memorials
to the authorities in South Africa and in Britain hoping that once the authorities were informed of
the plight of Indians, they would take sincere steps to redress their grievances as the Indians were,
after all, British subjects. To unite different sections of Indians, residing in South Africa, he set up
the Natal Indian Congress and started a paper Indian Opinion. So, statement 1 is correct.

New legislation in South Africa made it compulsory for Indians there to carry at all times certificates
of registration with their fingerprints. The Indians under Gandhi’s leadership decided not to submit
to this discriminatory measure. Gandhi formed the Passive Resistance Association to conduct the
campaign of defying the law and suffering all the penalties resulting from such defiance. Thus was
born satyagraha or devotion to truth, the technique of resisting adversaries without violence. So,
statement 2 is correct

The Tolstoy Farm was founded in 1910 . Besides being an experiment in education, it was to house
the families of satyagrahis and to give them a way to sustain themselves. So, statement 3 is
correct.

Q.40) Consider the following statements regarding Swaraj party formed in 1922:

1. C.R Das, Motilal Nehru and Ajmal Khan were the founding members of the Swaraj
party.

2. It called for repeal of repressive laws in Legislative councils.

3. The proposal of the Swaraj party for council entry was given congress approval in the
1922 Gaya session.

Which of the above given statements is/are correct?

(a) 1 and 2 only

(b) 2 and 3 only

(c) 1 only

(d) 3 only
EXPLANATION

The founding members of the Swaraj party are C.R. Dass, Motilal Nehru, and Ajmal, al Khan who
wanted to end the boycott of the legislative council so that nationalists could enter them to expose the
basic weakness of these assemblies and use these councils as an arena of political struggle to arouse
popular enthusiasm. So, statement 1 is correct.
They continuously outvoted the government on several issues including budgetary grants.The
Government was defeated several times on the question of the repeal of repressive laws and regulations
and the release of political prisoners.They defeated the Government on a number of bills. Noteworthy
was the defeat of the Government on the Public Safety Bill in 1928, through which the Government
proposed to acquire the power to deport undesirable and subversive foreigners. So, statement 2 is
correct.
The proposal for the entry of council by the swaraj party was defeated in the Gaya session of Congress
in 1922. So, statement 3 is not correct.

Q.41) Consider the following statements:

1. The communist party of India was formed in 1920 in Tashkent by M.N Roy and Abani
Mukherji.

2. S.A Dange and Muzaffar Ahmed were among the communist leaders who were jailed in
the Meerut conspiracy case.

3. Subash Chandra Bose became the President of All Bengal Student’s Conference in
1928.

Which of the above given statements is/are correct?

(a) 1 only

(b) 2 only

(c) 3 only

(d) None

EXPLANATION

The Communist Party of India (CPI) was formed in 1920 in Tashkent (now, the capital of Uzbekistan)
by M.N. Roy, Abani Mukherji, and others after the second Congress of Comintern. M.N. Roy was also
the first to be elected to the leadership of Commintern. So, statement 1 is correct.

In 1924, many communists—S.A. Dange, Muzaffar Ahmed, Shaukat Usmani, Nalini Gupta were jailed
in the Kanpur Bolshevik Conspiracy Case. In 1925, the Indian Communist Conference at Kanpur
formalized the foundation of the CPI.

In 1929, the government crackdown on communists resulted in the arrest and trial of 31 leading
communists, trade unionists, and left-wing leaders; they have tried at Meerut in the famous Meerut
conspiracy case. So, statement 2 is not correct.

The first All-Bengal Student’s Conference of Students was held in August 1928 and was presided over
by Jawaharlal Nehru (not by Subash Chandra Bose). So, statement 3 is not correct.

Q.42) In the context of Indian freedom struggle, which of the following events happened
earliest?

(a) Kakori robbery

(b) Bomb in central legislative assembly

(c) Saunder’s murder

(d) Formation of Hindustan Republican Army

EXPLANATION

Kakori Robbery - The most important action of the HRA was the Kakori robbery in August 1925.
The men held up the 8-Down train at Kakori, an obscure village near Lucknow, and looted its
official railway cash. Government crackdown after the Kakori robbery led to arrests of many, of
whom 17 were jailed, four transported for life and four—Bismil, Ashfaqullah, Roshan Singh, and
Rajendra Lahiri—were hanged. Kakori proved to be a setback.

Formation of the Hindustan Republican Army - Hindustan Republican Association (HRA) was
a revolutionary organization of India established in 1924 at village Bholachang in East Bengal by
Sachindra Nath Sanyal, Narendra Mohan Sen, and Pratul Ganguly as an offshoot of Anushilan
Samiti. Its objective was to establish a “Federated Republic of the United States of India” through
an organized and armed revolution.

Saunders’ Murder - (Lahore, December 1928) Just when the HSRA revolutionaries had begun to
move away from individual heroic action, the death of Sher-i-Punjab Lala Lajpat Rai due to lathi
blows received during a lathi charge on an anti-Simon Commission procession (October 1928) Jed
them once again to take to the individual assassination. Bhagat Singh, Azad, and Rajguru shot
dead Saunders, the police official responsible for the lathicharge in Lahore.

Bomb in the central legislative assembly ( April 1929)– The HSRA leadership now decided to
let the people know about its changed objectives and the need for a revolution by the masses.
Bhagat Singh and Batukeshwar Dutt were asked to throw a bomb in the Central Legislative
Assembly on April 8, 1929, to protest against the passage of the Public Safety Bill and Trade
Disputes Bill aimed at curtailing civil liberties of citizens in general and workers in particular, The
bombs had been deliberately made harmless and were aimed at making ‘the deaf hear’. The
objective was to get arrested and to use the trial court as a forum for propaganda so that people
Would become familiar with their movement and ideology.
Earliest happened incident was the founding of Hindustan Republic Army. So, option (d) is
correct

Q.43) Which of the following statements is/are correct with respect to ‘Delhi Proposals’
which were put forward in response to the Nehru report?

1. It proposed separate electorates for muslims in place of reserved seats proposed by the
Nehru report.

2. It demanded one third representation to muslims in the central legislative assembly.

3. The proposals asked for creation of three new Muslim majority provinces of Sinfh,
Baluchistan and North-West Frontier province.

Select the correct answer using the code given below:

(a) 1 and 2 only

(b) 2 and 3 only

(c) 1 and 3 only

(d) 1,2 and 3

EXPLANATION

December 1927, a large number of Muslim leaders had met at Delhi at the Muslim League session
and evolved - four proposals for their demands to be incorporated into the draft constitution. These
proposals, which were accepted by - the Madras session of the Congress (December 1927), came to
be known as the Delhi Proposals’. These were:

 It proposed a joint electorate in place of a separate electorate with reserved seats for
Muslims.. So, statement 1 is not correct

 one-third representation to Muslims in Central Legislative Assembly. So, statement 2 is


correct.
 representation to Muslims in Punjab and Bengal in proportion to their population;

 formation of three new Muslim majority provinces Sindh, Baluchistan, and North-West
Frontier Province. So, statement 3 is correct.
Q.44) Consider the following statements:

1. The Nehru report was prepared by Indian leaders in response to the challenge put
forward by Lord Birkenhead.

2. Mahatma Gandhi did not participate in any of the round table conferences convened by
the Britishers.

3. B.R Ambedkar and Tej Bahadur Sapru participated in all the three round table
conferences.

Which of the above given statements is/are correct?

(a) 1 only

(b) 2 and 3 only

(c) 1 and 3 only

(d) 1,2 and 3

EXPLANATION

As an answer to Lord Birkenhead’s challenge, an All Parties Conference met in February 1928 and
appointed a sub committee under the chairmanship of Motilal Nehru to draft a constitution.’ This
was the first major attempt by the Indians to draft a constitutional framework for the country. The
committee included Tej Bahadur Sapru, Subhash Bose, M.S. Aney, Mangal Singh, Ali Imam,
Shuab Qureshi, and G.R. Pradhan as its members. The report was finalized by August 1928. So,
statement 1 is correct.

The second round table conferences were held in London from September 1931 to December 1931.
The INC nominated Gandhi as the soul representative. Thus Gandhi attended the 2 nd Round table
conference. In 1st and 3rd, RTC Gandhi was not present. So, statement 2 is not correct.

B.R Ambedkar represented the Depressed class and Tej Bahadur Sapru represented the Indian
Liberal Party. Both B.R Ambedkar and Tej Bahadur Sapru participated in all the three round table
conferences. So, statement 3 is correct.

Q.45) Which of the following movements/protests were inspired by the Civil


Disobedience movement started by Mahatma Gandhi?

1. No tax movement in Gujarat

2. Protests against forest laws in Maharashtra, Karnataka and central provinces

3. Agitation against Cunningham circular

4. No rent campaign in U.P

Select the correct answer using the code given below:


(a) 1,2 and 3 only

(b) 2 and 3 only

(c) 1 and 4 only

(d) 1,2,3 and 4

EXPLANATION

All four movements were inspired by the Civil Disobedience Movement started by Mahatma Gandhi.

The Civil Disobedience Movement was carried in different parts of the country. The places include
Tamilnadu, Malabar, Andhra Region, Orissa, Gujarat, Maharashtra, Assam, Bihar, Peshawar,
Bengal, Sholapur, United Provinces, and Dharsana. So, option (d) is correct.

ADDITIONAL INFORMATION

Civil Disobidience Movement

No tax movement in Gujarat

 A determined no-tax movement was organized in Gujarat which included refusal to pay
land revenue.

 Villagers crossed the border into neighboring princely states (such as Baroda) with their
families and belongings and camped in the open for months to evade police repression.
The police retaliated by destroying their property and confiscating their land.

Protests against forest laws in Maharashtra, Karnataka, and central provinces

 Maharashtra, Karnataka, Central Provinces These areas saw defiance of forest laws
such as grazing and timber restrictions and public sale of illegally acquired forest
produce.

Agitation against Cunningham circular

 A successful students strike was organized against the Cunningham circular which
banned student's participation in politics.

No rent campaign in U.P

 A no-revenue campaign was organized; a call was given to zamindars to refuse to pay
revenue to the government.

 Under a no-rent campaign, a call was given to tenants against zamindars. Since most of
the zamindars were loyalists, the campaign became virtually a no–rent campaign.
Q.46) Consider the following pairs:

Newspapers Founder/Editor

1. Bengal Gazette - Harishchandra Ray

2. Indian Mirror - Annie Besant

3. Amrita Bazar - Sisir Kumar Ghosh

Patrika

4. Swadeshamitram- Girishchandra Ghosh

Which of the above given pairs is/are correct?

(a) 1 and 3 only

(b) 2 and 3 only

(c) 1 and 4 only

(d) 2 and 4 only

EXPLANATION

1. Bengal Gazette - It was the first Bengali newspaper founded by Harishchandra


Ray in 1818.

2. Indian Mirror – It was founded by Denvendranath Tagore and NN Sen in 1862.

3. Amrita Bazar – It was founded by Sisir Kumar Ghosh. It was a Bengali


newspaper.

4. Swadeshamitram - It was a newspaper founded by G Subramania Ayer in


Madras.

So, option (a) is correct.

Q.47) Consider the following statements regarding Cripps Mission;

1. The main purpose of the mission was to seek Indian support for the Allies in the second
world war.

2. The mission proposed an Indian union with dominion status and a constituent
assembly to frame a new Constitution.

3. The mission did away with the right of any province to secede from the Indian union.

Which of the above given statements is/are correct?

(a) 1 only

(b) 1 and 2 only


(c) 2 and 3 only

(d) 1,2 and 3

EXPLANATION

The proposal of the mission was due to the reverses suffered by the British in South East Asia, in
Second world war. The Japanese threat to invade India seemed real now and India's support
became crucial. There was also pressure from America to Britain to seek Indian Cooperation. So
British Cripps's proposal was to seek Indian support for the Allies in the Second World war. So,
statement 1 is correct.

Tue Mission proposed an Indian Union with dominion status would be established and after the
end of the war, a Constituent Assembly would be convened to frame a new Constitution. Members
of the Assembly would be partly elected by the provincial assemblies through proportional
representation and partly nominated by the princes. So, statement 2 is correct.

According to Cripps Mission any province unwilling to join the Indian dominion could form a
separate union and have a separate constitution.The right to secede from the Indian Union
completely went against the principle of national unity.So, statement 3 is not correct.

Q.48) Consider the following statements regarding Subash Chandra Bose:

1. Subash Chandra Bose founded the Independence league in 1927.

2. He founded the ‘Swaraj’ newspaper and was also the editor of ‘Forward’ newspaper
founded by C.R Das.

3. He formed the provincial government for free India in 1943 at Singapore.

Which of the above given statements is/are correct?

(a) 1 and 2 only

(b) 2 and 3 only

(c) 1 and 3 only

(d) 1,2 and 3

EXPLANATION

Jawaharlal Nehru along with Srinivasa Iyengar and Subhash Chandra Bose founded the
Independence for India League in the year 1928 which put forth the demand for complete
independence. Srinivasa Iyengar was its first president. So, statement 1 is not correct.

He started the newspaper Swaraj and took charge of publicity for the Bengal Provincial Congress
Committee. He was also the editor of the newspaper "Forward", founded by Chittaranjan Das. So,
statement 2 is correct.

On 21 October 1943, Bose announced the formation of the Provisional Government of Azad Hind
(Free India) at Singapore, with himself as the Head of State, Prime Minister, and Minister of War.

The Provisional Government not only enabled Bose to negotiate with the Japanese on an equal
footing but also facilitated the mobilization of Indians in East Asia to join and support the INA. Soon
after the announcement, the Provisional Government received recognition from various countries.
So, statement 3 is correct.

Q.49) Which of the following organisations were established by D.R B.R Ambedkar?

1. All India Scheduled Caste Federation

2. Independent Labour Party

3. All India Anti Untouchability League

Select the correct answer using the code given below:

(a) 1 and 2 only

(b) 2 and 3 only

(c) 2 only

(d) 1 and 3 only

EXPLANATION

The following organisations were established by D.R B.R Ambedkar.

1. Independent Labour Party - Ambedkar Independent Labour Party (ILP) formed in 1936 with a
comprehensive programme to meet the needs and grievances of the landless, poor tenants,
agriculturists, and workers. In the polls held in 1937, the first election under the newly
enacted Government of India Act of 1935, the ILP achieved spectacular success by winning
15 of the 17 seats it had contested for the Bombay Legislative Assembly.

2. All India Scheduled Caste Federation - In 1942 Ambedkar established All India Scheduled
Caste Federation to solve the problems of Dalits .

All India Anti Untouchability League was formed by M. K . Gandhi. So option (a) is correct.
Q.50) Which of the following missions/plan of British government was described by
Mahatma Gandhi as ‘ a post dated cheque’ ?

(a) Wavell plan

(b) August offer

(c) Cripps mission

(d) Cabinet mission

EXPLANATION

Gandhi describes Cripps mission as ‘a post dated cheque’, because British were losing their hold,
so the proposal of granting the subcontinent a dominion status through Cripps mission was like a
post-dated cheque in a bank which is going to fail.

The Cripps Mission proposals failed to satisfy Indian nationalists and turned out to be merely a
propaganda device. Various parties and groups had objections to the proposals on different points.
The Congress objected to:
 the offer of dominion status instead of a provision for complete independence;
 representation of the princely states by nominees and not by elected representatives;
 right to provinces to secede as this went against the principle of national unity; and

 absence of any plan for immediate transfer of power and absence of any real share in
defence; the governor-general’s supremacy had been retained, and the demand that the
governor-general be only the constitutional head had not been accepted. So option (c) is
correct.

Q.51) With reference to foreign travellers visit to India, arrange the following in the
correct chronological order of their arrival in India:

1. Al- Biruni

2. Ibn Batuta

3. Francois Bernier

4. Abd-al-Razzāq Samarqandī

Select the correct answer using the code given below:

(a) 1-2-3-4

(b) 1-2-4-3

(c) 2-1-3-4

(d) 3-1-2-4
EXPLANATION

The chronological order will be

Al- Biruni came to India in Eleventh century. Al-Beruni came to India during the period of Sultan
of Muhammad of Ghazni.

Ibn Batuta came to India in Fourteenth century. A Moroccan traveler, Ibn Battuta (1333-1347 AD)
visited India during the reign of Muhammad-bin-Tughlaqp.

Abd-al-Razzāq Samarqandī visited India in Fifteenth century. He visited the city of Vijayanagara.

Francois Bernier visited India in Seventeenth century. He was the French physician and traveller
who reached India in the seventeenth century, towards the end of 1658 A.D. at the beginning of
Aurangzeb's reign. So option (b) is correct.

Q.52) In the context of the history of medieval India, the term ‘uluq’ and ‘dawa’ were
used in the context of?

(a) Postal system of communication

(b) System of land revenue collection

(c) Caste division in the medieval society

(d) System of medicine used for treatment

EXPLANATION

The term ‘uluq’ and ‘dawa’ were used in the context of a Postal system of communication.
In India the postal system is of two kinds.

 The horse post, called uluq is run by royal horses stationed at a distance of every four
miles. The foot-post has three stations per mile; it is called dawa, that is one-third of a
mile .

 Now, at every third of a mile there is a well populated village, outside which are three
pavilions in which sit men ready to start.

 Each of them carries a rod, two cubits in length, with copper bells at the top. When the
courier starts from the city he holds the letter in one hand and the rod with its bells on
the other; and he runs as fast as he can.

 When the men in the pavilion hear the ringing of the bell they get ready.

 As soon as the courier reaches them, one of them takes the letter from his hand and
runs at top speed shaking the rod all the while until he reaches the next dawa.

 And the same process continues till the letter reaches its destination. This foot-post is
quicker than the horse-post; and often it is used to transport the fruits of Khurasan
which are much desired in India.

Q.53) Of all the accounts of the foreign travellers who visited India during the sixteenth
and seventeenth century, the most common description by all the travellers is?

(a) Presence of a socially stratified caste system

(b) Well established system of trading with foreign countries

(c) Lack of private property in land and crown ownership of land

(d) Presence of widespread poverty among Indians

EXPLANATION

According to Bernier, one of the fundamental differences between Mughal India and Europe was
the lack of private property in land in the former. He was a firm believer in the virtues of private
property, and saw crown ownership of land as being harmful for both the state and its people.
He thought that in the Mughal Empire the emperor owned all the land and distributed it among
his nobles, and this had disastrous consequences for the economy and society. This perception
was not unique to Bernier, but is found in most travelers accounts of the sixteenth and
seventeenth centuries. So statement (c) is correct.

Q.54) Consider the following pairs:

Stages of Harappan Site

Civilization

1. Pre-Harappan - Mohenjo Daro

2. Early- Harappan - Kalibangan

3. Mature Harappan - Kot Diji

4. Late Harappan - Lothal

Which of the above given pairs is/are correctly matched?

(a) 1 and 2 only

(b) 2 and 3 only

(c) 1 and 4 only

(d) 3 and 4 only


EXPLANATION

1. Pre-Harappan - Site which belongs to Pre-Harappan Stage is Mohenjo Daro.

2. Early- Harappan – The Site which belongs to Early- Harappan Stage is Kot Digi.

3. Mature Harappan - The Site which belongs to Mature- Harappan Stage is Kalibangan.

4. Late – Harappan - The Site which belongs to Late- Harappan Stage is Lothal.

Hence 1 and 4 are correctly matched. So, option (c) is correct.

Q.55) With reference to socio- cultural and economic life of Indus valley civilization,
consider the following statements:

1. The large scale use of burnt bricks and total absence of stone buildings was a unique
feature of urban centres.

2. No evidence of the practice of the sati system can be found in the Harappan civilization.

3. Gold, copper and iron were the most used metals by the Indus valley people.

Which of the above given statements is/are correct?

(a) 1 and 2 only

(b) 1 only

(c) 2 and 3 only

(d) 1 and 3 only

EXPLANATION

In Indus Valley civilization burnt bricks were extensively used for construction purposes of Houses
and drainage system and there is complete absence of stone buildings which was the unique
feature of urban centres. So, Statement 1 is correct.

Most archaeological recoveries show individuals were buried separately in Harappan times.
Joint graves have been very rare, and almost none have been found containing a couple.
 The only joint burial of a couple discovered earlier has been from Lothal in Gujarat.
 But there, the skeletal remains of the male and female were found placed over one other,
indicating that they may have been buried at the same place, but at different times.
 In addition, the female skeletal remains were found to have lesions or injury marks. This
has made archaeologists to conclude that her death could have been the result of a social
practice such as Sati.
So, Statement 2 is not correct.
The most used metals by the Indus valley people are bronze, copper, silver and gold. Iron was not
used by the Indus valley people. Evidence also shows the utilisation of lead alloy with copper and
bronze in Indus valley (2500 B.C.). Indus Valley Civilisation also got the credit for the development
of various alloys in the south-Asian countries. They used to make alloys such as bronze, Tin and
arsenic bronze by smelting copper with other metals. Bronze was extensively used in making
sculptures. So, Statement 3 is not correct.

Q.56) Consider the following statements with regard to Stupas:

1. The practice of stupa building started during the Mauryan rule under Ashoka.

2. The core of the stupa was made of unburnt brick and the outer surface was decorated
with stone structures.

3. Piprahwa stupa in Uttarpradesh is the oldest stupa in India.

Which of the above given statements is/are correct?

(a) 1 and 2 only

(b) 3 only

(c) 1 and 3 only

(d) 2 and 3 only

EXPLANATION

The construction of stupas was practiced from Vedic period. However Stupas for Buddha, on a large
scale and associated with Buddhism, began throughout India during the reign of Ashoka the Great
(268-232 BCE) of the Mauryan Empire, after his conversion to Buddhism. So statement 1 is not
correct.

The core of the stupa was made up of unburnt brick and the outer face of burnt brick, covered with
thick layer of plaster. The stupa was crowned by an umbrella of wooden fence enclosing a path for
pradaskshina (circumambulatory path). One of the best examples of the structure of a stupa in the
third century BCE is at Bairat in Rajasthan. So statement 2 is not correct.

Piprahwa stupa in Uttar Pradesh is the oldest stupa in India. Piprahwa is a village near Birdpur in
Siddharthnagar district of the Indian state of Uttar Pradesh. Piprahwa is best known for its
archaeological site and excavations that suggest that it may have been the burial place of the portion of
the Buddha’s ashes that were given to his own Sakya clan. So statement 3 is correct.
ADDITIONAL INFORMATION

Stupas

 The stupa (a Sanskrit word meaning a heap) originated as a simple semi-circular


mound of earth, later called anda. Gradually, it evolved into a more complex structure,
balancing round and square shapes. Above the anda was the harmika, a balcony- like
structure that represented the abode of the gods.

 The early stupas at Sanchi and Bharhut were plain except for the stone railings, which
resembled a bamboo or wooden fence, and the gateways, which were richly carved and
installed at the four cardinal points.

 Worshippers entered through the eastern gateway and walked around the mound in a
clockwise direction keeping the mound on the right, imitating the sun’s course through
the sky. Later, the mound of the stupas came to be elaborately carved with niches and
sculptures as at Amaravati, and Shah- ji-ki-Dheri in Peshawar (Pakistan).

Q.57) Consider the following statements:

1. The Mauryan period saw the emergence of rock-cut cave architectures in India.

2. Yaksha and Yashi were the Mauryan sculptures which were also the objects of worship
for Jainism, Buddhism and Hinduism.

3. Chaitya halls were primarily used for offering prayers during the post Mauryan period.

Which of the above given statements is/are correct?

(a) 1 and 2 only

(b) 2 and 3 only

(c) 1 and 3 only

(d) 1,2 and 3

EXPLANATION

During the Mauryan period the rock cut architecture was evolved. The artisans in the Mauryan
started the practice of hewing out caves from rocks for monks to live in. The earliest examples
are the Barbara caves at distance of 30km from Gaya. So statement 1 is correct.

Famous sculptures of the Mauryan period are those of Yaksha and Yakshi. They were objects of
worship related to all three religions – Jainism, Hinduism, and Buddhism. The earliest mention
of yakshi can be found in Silappadikaram, a Tamil text. So statement 2 is correct.
Post Mauryan period saw the emergence of Chaitya and Viharas. Chaitya was mostly used for
offering prayers. The front of the chaitya hall is dominated by the motif of a semi-circular chaitya
arch with an open front which has a wooden façade and, in some cases; there is no dominating
chaitya arch window. In all the chaitya caves, a stupa at the back is common. So statement 3 is
correct.

Q.58) Consider the following statements with regard to Ajanta caves:

1. The Ajanta caves were developed under the patronage of Vakataka king Harishena.

2. The paintings in the caves are mainly dedicated to the life of Buddha and jataka stories.

3. Presence of blue colour in the paintings was a unique feature of Ajanta caves.

Which of the above given statements is/are correct?

(a) 1 and 2 only

(b) 1 only

(c) 2 and 3 only

(d) 1,2 and 3

EXPLANATION

Ajanta caves saw the development in 200 B.C 650 A.D. Tue Ajanta caves was mostly developed
by the Buddhist monks under the patronage of Vakataka king Harishena. Ajanta has 29 caves.
These caves were under UNESCO world heritage site which was designated in 1983. So
statement 1 is correct.

The paintings in the Ajanta caves are generally themed around Buddhism, the life of Buddha
and Jataka stories. So statement 2 is correct.

The unique feature of the Ajanta caves is that complete absence of Blue colour in the painting.
The outlines of the paintings were mostly done in the red colour. So statement 3 is not correct.

Q.59) The ‘Vajra Mudra’ of Buddha signifies?

(a) Teaching phase of preaching in Buddhism

(b) Supreme enlightenment and perfection

(c) Importance of knowledge or supreme wisdom

(d) Strength and inner security

EXPLANATION

Vajra Mudra signifies the importance of knowledge or supreme wisdom. Knowledge is represented by
the forefinger and the fist of the right hand protects it. This Mudra is better known in Korea and
Japan.

So option (c) is correct.

Q.60) ‘The cave was created in the early 5th century AD under the patronage of Gupta
rulers. It is famous for having numerous sculptures with the sculpture of Varaha and
Boar incarnation of Vishnu being the famous ones. The cave also has one of the
earliest Hindu sculptures dedicated to shiva, Narasimha, Narayana.’

Identify the architecture being described in the above passage:

(a) Bagh caves

(b) Udayagiri caves

(c) Junagadh caves

(d) Ellora caves


EXPLANATION

 Udayagiri Caves is a group of 20 Gupta-era temples and monasteries carved out of a rocky
hill, out of which one is dedicated to Jainism and rest to Hinduism.
 Brahmi inscriptions on the caves indicate that the site was excavated during the rule of
Gupta King, Chandragupta II (AD 376-413).
 Cave 5, also known as the Varaha Cave, depicts Vishnu in a massive carving as Varaha
(boar) incarnation and rescuing goddess Earth (Bhudevi, Prithivi) from the depths of cosmic
ocean.
 Varaha, as embedded in Hindu text, is a symbol of right versus wrong and good versus evil.
 The doorway of Cave 6 has ornately carved figures of Ganesha, Vishnu and Shiva on the left
and Brahma, Vishnu and Shakti Durga as Mahishasura Mardini on the right.
 Cave 13 has a Anantasayana panel featuring a 12-ft long, remarkable rock-cut sculpture of a
reclining Vishnu.
 Situated in Sonpura and Udayagiri villages of Madhya Pradesh, these caves are the finest
example of classical Gupta art.
 The caves have one of the earliest Hindu sculptures. It also has caves dedicated to Shiva,
Narasimha (half-lion, half-man), Narayana (resting Vishnu) and Skanda.
So option (b) is correct.

Q.61) Consider the following statements with regard to development of science during
gupta period:

1. Aryabhatta belonged to the Gupta period and he was the first to declare that the earth
is spherical and rotates on its own axis.

2. Varahmira composed Panch Siddhanta and wrote Brihadsamhita.

3. Vagbhata, Charaka and Susruta were the medical trio who all belonged to the Gupta
period.

Which of the above given statements is/are correct?

(a) 1 and 2 only

(b) 2 only

(c) 1 and 3 only

(d) 1,2 and 3

EXPLANATION

Aryabhatta was one of the greatest scholars during the period of Gupta. He first envisioned the
concept of Zero and he was the first mathematician- astronomer who declared the earth to be
spherical and rotates on its own axis. So, statement 1 is correct.

Varahmira composed two books that were famous around the world and translated into different
languages.

1. Panch Siddhanta

2. Brihadsamhita

So, statement 2 is correct.

Vagbhata is one of the greatest physicians and the last medical trio of the Gupta period. Vagbhata
was also author Ashtangasamgraha which explains the Summary of the eight branches of
medicine. The other two are Charaka and Susruta were lived before the Gupta Period. So,
statement 3 is not correct.

Q.62) Consider the following literary works of ancient India:

1. Malavikagnimitra

2. Mudrarakshasa

3. Raghuvamsa

4. Mrichchhakatika

Which of the above were the literary works of Kalidasa?

(a) 1,2 and 3 only

(b) 2 and 4 only

(c) 1 and 3 only

(d) 3 and 4 only


EXPLANATION

Following are the some of the works done by the Kalidasa

1. Malavikagnimitra - It concerns the machinations of King Agnimitra to win Malavika, a


female dance student with whom he is in love.

2. Raghuvamsa - The play traces the roots of the great lineage of Lord Rama and his
descendants and the great conqueror Raghu. Raghuvamsam talks about the valor and
strength of the great warrior Raghu. It is the story of some of the great warrior kings ever
born on Indian soil.

3. Kumara sambhava - Kumarasambhava describes the birth and adolescence of the goddess
Parvati, and her marriage with Lord Shiva.

4. Ritusamhara - describes the six seasons by narrating the experiences of two lovers in each
of the seasons.

5. Meghaduta - describes the story of a Yaksha trying to send a message to his lover through
a cloud. This poem is elegiac in nature through which Kalidasa created his own genre of
poetry.

Mrichchhakatika and Mudrarakshasa are the literary works of Vishakhadatta.

So, option (c) is correct.

Q.63) Consider the following statements with regard to Aihole inscription:

1. The inscription is written in Sanskrit and it uses the Kannada script.

2. It mentions the defeat of Harshsvarshana by king Pulkeshin II and also the victory of
Chalukyas over pallavas.

Which of the above given statements is/are correct?

(a) 1 only

(b) 2 only

(c) Both 1 and 2

(d) Neither 1 nor 2

EXPLANATION

The Aihole Inscription of Ravi Kirti at the Meguti temple, is in Sanskrit language and old Kannada
script. It is considered one of the finest pieces of extant poetry in the Sanskrit language and is a
eulogy of King Pulakesi and his conquests. So, statement 1 is correct.
In the Aihole Inscription there is a mention about the defeat of Harshavardhana by Pulakesin II and
also about the victory of Chalukyas on Pallavas. There is also a reference to the shifting of the
capital from Aihole to Badami by Pulakesin II. So, statement 2 is correct.

ADDITIONAL INFORMATION

AIHOLE

 Aihole is a western Chalukyas site that is situated in Karnataka.

 It was built between 450 and 650 C.E. that is situated on the banks of the river
Malaprabha with the Meguti hill overlooking it.

 It was known in ancient India as Aryapur or Ayyavole and was probably the earliest
capital of the Chalukyas.

 For two centuries it was at the center of the temple building.

 There are over a hundred temples in Aihole, all marking the shift from rock-cut temple
architecture to a form where stone blocks are placed on top of one another.

 The oldest temple in Aihole is the Lad Khan temple.

 The Aihole Inscription is a great source for reconstructing the history of the period and
it records the achievements of Pulakesi II

 Monuments belonging to all the three religions –Hinduism, Buddhism, and Jainism are
found here.

 Badami and Pattadakkal are other important chalukiyan sites in its vicinity.

Q.64) Which of the following is/are the features of Nagara style of temple architecture
in India?

1. Panchayatana style of temple making

2. Presence of water tanks in the temple premises

3. Covered ambulatory passageway or the Pradakshina Path around the sanctum


sanctorum

4. Presence of elaborate boundary walls or gateways.

Select the correct answer using the code given below:

(a) 1 and 2 only

(b) 1 and 3 only

(c) 2 and 4 only

(d) 3 and 4 only


EXPLANATION

The style of temple architecture that became popular in northern India is known as nagara. In
North India, it is common for an entire temple to be built on a stone platform with steps leading up
to it.

These are the common feature of the Nagar style of temple making

1. It is in Panchayatana style - the main shrine built on a rectangular plinth of four smaller
subsidiary shrines at the four corners.

2. Pradakshina's path around the sanctum, Sanctorum, and The ambulatory path around the
Garbhagriha are covered.

3. The entire temple is built on a single stone platform with steps leading up to it.

There is no presence of water tanks in the Temple premises, the presence of a water tank and
presence of elaborate boundary walls or gateways are the feature of Dravida or South Indian style
of Temple.

So, option (b) is correct.

Q.65) Which of the following schools of sculpture has influence of all the three religions
namely Hinduism, Jainism and Buddhism?

(a) Gandhara school

(b) Mathura school

(c) Amravati school

(d) Both b and c

EXPLANATION

Mathura school of art influences all the three religions such as Hinduism, Buddhism, and
Jainism.

 The Mathura School flourished on the banks of the river Yamuna in the period between
the 1st and 3rd centuries B.C.
 In Mathura, an indigenous style of sculpture developed, and it mostly used red
sandstone.
 Images of Vaishnava (mainly Vishnu and his various forms) and Shaiva (mainly the
lingas and mukhalingas) faiths are also found at Mathura but Buddhist images are
found in large numbers. It may be noted that the images of Vishnu and Shiva are
represented by their ayudhas (weapons).

 The sculptures of the Mathura School were influenced by the stories and images of all
three religions of the time – Buddhism, Hinduism, and Jainism.

Gandhara school was mainly influenced by Buddhism. Amaravati school also mainly influenced
the Buddhists. So, option (b) is correct.

Q.66) Which of the following is not a part of the ‘Group of monuments at


Mahabalipuram’?

(a) Pancha Ratha

(b) Open air rock reliefs

(c) Shore temple complex

(d) Hathigumpha inscriptions

EXPLANATION

This group of monuments, founded by the Pallava kings, was carved out of rock along the
Coromandel coast in the 7th and 8th centuries. It has been listed under UNESCO World’s
Heritage Site.

It is known especially for its

 rathas (temples in the form of chariots) - The five rathas that can be seen are
Dharmaraja Ratha, Bhima Ratha, Arjuna Ratha, Draupadi Ratha, and Nakul Sahadev
Ratha.

 mandapas (cave sanctuaries),

 giant open-air reliefs such as the famous 'Descent of the Ganges', and

 the temple of Rivage, with thousands of sculptures to the glory of Shiva.

 Shore temple complex

Hathigumpha inscriptions are not a part of monuments at Mahabalipuram. So, option (d) is
correct.

Q.67) With reference to jain pilgrimage sites in India, consider the following pairs:

Sites Place

1. Pawapuri - Nalanda

2. Dilwara temples - Mount Abu

3. Kankali Tila - Tahrabad in

Maharashtra
4. Mangi- Tungi - Mathura

Which of the above given pairs is/are correctly matched?

(a) 1 and 2 only

(b) 2 and 4 only

(c) 2 and 3 only

(d) 1 and 4 only

EXPLANATION

Pawapuri - The Jal Mandir meaning Water Temple, also known as Apapuri, in Pawapuri, meaning
a town without sins, in the district of Nalanda, the Indian state of Bihar, is a highly revered
temple dedicated to Lord Mahavira, the 24th Thirthankara (religious preacher of Jainism) and
founder of Jain religion, which marks the place of his cremation. Mahavira attained Nirvana
(salvation) in Pawapuri in 528 BC. The temple has been built within a tank filled with red-colored
lotus flowers. It is said that the temple was built by King Nandivardhan, Mahavira’s elder brother.
It is one of the five main temples in Pawpuri, where the “Charan Paduka” or foot impression of
Mahavira is deified.

Dilwara temples - Dilwara Jain Temple is one of the world's most beautiful and sacred pilgrimage
of Jains in the world. Located just 2.5 km from Mount Abu, Rajasthan. Built-in between the 11th
and 13th centuries AD by the Chalukya dynasty, Dilwara Jain temples are simple and general
looks from the outside but its extravagant doors of entrance tell about its architectural superiority.
It is surrounded by green hills and a temple complex with high walls.

Kankali Tila - Kankali Tila is a mound located at Mathura deriving Its name from a temple of
Hindu goddess Kankall which was erected in its vicinity In recent times ,3,000-year ancient Jain
stupa was excavated here from 1871-1896.

Mangi – Tungi - Mngi Tungi Temple is 125 km from Nashik, near Tahrabad Maharashtra. It is
associated with Jainism and also Hindhusim.

1 and 2 are correctly matched.


So, option (a) is correct.

Q.68) In the context of the ancient history of India, ‘Basadis’ were used to denote?

(a) Buddhist pilgrimage sites

(b) Resting place for monks

(c) Jain temples

(d) Centers for barter exchange


EXPLANATION

The Basadis in ancient history are used to denote Jain temples. So, option (c) is correct.

Q.69) Consider the following statements:

1. While Trabeat style of temple construction uses lintel, the Arcuade style used arches
and domes.

2. Minars were present in both the Trabeat and the Arcuade style.

3. Stone was primarily used in Trabeat style while bricks and mortar were chiefly used in
Arcuade style of temple building.

Which of the above given statements is/are correct?

(a) 1 and 2 only

(b) 2 and 3 only

(c) 1 and 3 only

(d) 1,2 and 3

EXPLANATION

Trabeat style of temple construction uses lintel – using horizontal beam across two vertical
columns. Arcude style of temple construction used dome in top and arches in the entrance.
Arcuate style architecture replaced the Trabeat style as arches and domes were introduced. So,
statement 1 is correct

Minars were absent in Trabeat and Minars were present on Arcuade style. So, statement 2 is
not correct.

Stones are the primary component for all the construction in the Trabeat Style while in the
Arcuade style bricks, like plaster and mortar were widely used. Mortar was used as a cementing
agent during construction. So, statement 3 is correct.

Q.70) ‘This temple is significant to the Hindu sects of both shaivism and Shaktism. The
inscriptional evidence suggests that the temple belonged to the Satavahana dynasty
and some further additions were also done during the Vijayanagar period. It is the
only temple in India which is renowned as both Jyotirlinga and Shaktipeeth.’

Identify the name of the temple from the above given description:
(a) Venkateshwara temple

(b) Veerabhadra temple

(c) Sri Bramarambha Mallikarjuna temple

(d) Virupaksha temple

EXPLANATION

Sri Bhramaramba Mallikarjuna Temple is a Hindu temple dedicated to the deities Shiva and Parvati,
located at Srisailam in the Indian state of Andhra Pradesh. It is the only temple in India that is
renowned as both Jyotirlinga and Shaktipeeth. There is inscriptional evidence from the Satavahana
dynasty which places the temple to be existent from the 2nd century. Most modern additions were
done during the time of king Harihara of Vyayanagara Empire.

So, option (c) is correct.

Q.71) Among the following famous universities of ancient India, which of the following
were primarily dedicated to learning Buddhism?

1. Vikramshila

2. Odantapuri

3. Takshashila

4. Kancheepuram

Select the correct answer using the code given below:

(a) 1 and 3 only

(b) 1 and 2 only

(c) 2 and 4 only

(d) 3 and 4 only


EXPLANATION

Takshashila was a noted center of learning, including religious teachings of Buddhism, for
several centuries. It continued to attract students from around the world until its destruction in
the 5th century CE. It is considered a place of religious and historical sanctity by Hindus
and Buddhists and was the seat of Vedic learning where the emperor Chandragupta
Maurya was taken there by Chanakya to learn in the institution.

Vikramshila University was the major Buddhist learning center in ancient India. It was
founded by the Pala dynasty in the 8th century and remained one of the top universities or
Mahavihara till the end of the 12th century when it was destroyed by notorious Turkish invader
Bakhtiyar khilji along with another symbol of contemporary temples of education and learning
Nalanda and Odantpuri.
Odantapuri University located on Hiranya Prabat in Bihar sarif is also known as odantpura
vihar or odantapuri Buddhist Mahavira. Founded in the 8th century by emperor Gopala of the
Pala Dynasty, it flourished for 400 years till the 12th century. It was one of the sixth universities
in ancient India established primarily to propagate Buddhist learning and teachings.
Kancheepuram University is a religious education center for Hindu, Jainism, and Buddhists.
Vikramshila and Odantapuri universities are primarily dedicated to Buddhist Learning.
Where other universities like odantapuri and Kancheepuram University are dedicated to
Hindu, Vedic, Buddhist and Jainism Learnings.
So, option (b) is correct.

Q.72) With reference to the architectural features of city of Fatehpur sikri , consider the
following statements:

1. The buildings in the city of Fatehpur Sikri represent a unique blend of Hindu and
Persian styles.

2. The Buland Darwaza is a red sandstone structure which was built to commemorate
Akbar’s victory over Gujarat.

3. Ibadat Khana was used as a place where Akbar used to meet leaders of different
religions.

Which of the above given statements is/are correct?

(a) 1 and 2 only

(b) 2 and 3 only

(c) 1 and 3 only

(d) 1,2 and 3


EXPLANATION

The buildings in the Fatehpur Sikri are the best example of the culmination of Hindu and Muslim
architecture. Fatehpur Sikri Mosque is said to be a copy of the mosque in Mecca and has designs,
derived from Persian & Hindu architecture. So, statement 1 is correct

Most of the buildings in the Fatehpur Sikri are built by Redstone because of their availability in
neighboring areas of Fatehpur. The Buland Darwaza is a red sandstone structure that was built to
commemorate Akbar’s victory over Gujarat in 1601. An inscription on the central face of the Buland
Darwaza describes Akbar's religious openness. So, statement 2 is correct.

The House of Worship or the Ibadat Khana was established by Akbar (1542-1605 CE) for conducting
religious debates and discussions among leaders of different religions. So, statement 3 is correct.

Q.73) A Mural painting which depicts a scene of Shibi Jataka, where the King Shibi
offered his own flesh to save the pigeon can be found in?

(a) Ellora caves

(b) Ajanta caves

(c) Jogimara caves

(d) Sittanavasal caves

EXPLANATION

A Mural painting that depicts a scene of Shibi Jataka, where King Shibi offered his own flesh to
save the pigeon can be found in Ajanta cave. So, option (b) is correct.

So, option (b) is correct.


ADDITIONAL INFORMATION

ELLORA CAVES

 Ellora caves are another important site of cave architecture. It is located nearly 100
Kms away from Ajanta caves. It is a group of 34 caves — 17 Brahmanical 12 Buddhist
and 5 Jain.

 These set of caves were developed during the period between 5th and 11th centuries AD
(newer as compared to Ajanta Caves) by various guilds from Vidarbha, Karnataka and
Tamil Nadu. Hence, the caves reflect a natural diversity in terms of theme and
architectural styles.

JOGIMARA CAVES

 Jogimara caves also called as Bengra cave is a 2300 years old performance stage - one
of the oldest theaters in the world. according to legends during Rama, Sita's exile, Sita
stayed in these caves and thus named as Sita Bengra - "Residence of Sita".

 The best example of the Pre-Buddha paintings is the Jogimara cave. The paintings of
these caves have been dated from 300BC to as back as 1000BC. The roof of the cave
has some seven paintings which include the human figures, fish and elephants.

SITTANAVASAL CAVES

 The Sittannavasal cave is a rock-cut temple or monastery that’s also known as Arivar
Koil. It is part of a 2nd century complex of Jain temples located in the Sittannavasal
village of Pudukkottai district of Tamil Nadu. In Tamil, the word Sittannavasal literally
translates to ‘abode of the great saints’. The temple is famous in India for its
magnificent mural paintings dating back to the 7th century.

 The painting technique used is Fresco-secco and similar to that used in the Ajanta
Caves. The murals are painted on surfaces that have first been covered with lime
plaster (2.5mm thickness) before adding a thin layer (0.5mm) of thick lime wash. The
pigments used for painting were permanent mineral colours with white colour being
made from lime, black colour from lamp soot or charcoal, red from red ochre, yellow
from yellow ochre, green from Terre verre and blue from ultramarine or lapis lazuli.

Q.74) Consider the following pairs:

Mural paintings Place/Period

1. Bagh cave paintings - Madhya

Pradesh
2. Armamalai cave - Tamil Nadu

paintings

3. Ravan Chhaya rock - Andhra

Shelter Pradesh

4. Lepakshi paintings - Odisha

Which of the above given pairs is/are correctly matched?

(a) 1 and 2 only

(b) 2 and 4 only

(c) 1 and 3 only

(d) 3 and 4 only

EXPLANATION

1. Bagh cave paintings - Located on the bank of the Bagh river in Madhya Pradesh, Bagh
Caves is a group of 9 Buddhist caves developed around the 6th Century A.D.

2. Armamalai cave paintings - It is located in the Vellore district of Tamilnadu. Mud and
unbaked brick structure, located within the cave place were the resting places of Jain
saints. Beautiful color paintings are seen on the cave roof and walls of the structure.
These paintings depict the stories of Jainism and Astathik Palakas (eight corners) with
their mounts.

3. Ravan Chhaya rock shelter – Located in the Keonjhar district of Odisha. These are
ancient fresco paintings on a rock shelter are in the shape of a half-opened umbrella.
Believed that this shelter acted like the royal hunting lodge. The most noticeable
painting -a royal procession dates back to the 7th century.
4. Lepakshi paintings - Located in Anandpur district of Andhra Pradesh, these mural
paintings were executed on Veerabhadra temple walls at Lepakshi in 16th century. Made
during the Vijayanagara period, they follow a religious theme based on Ramayana,
Mahabharata and incarnations of Vishnu. The paintings show a complete absence of
primary colours, especially blue. They depict a decline in painting in terms of quality.
The forms, figures and details of their costumes are outlined with black colour.
1 and 2 are correctly matched. So, option (a) is correct.

Q.75) The ‘Tamasha Paintings’ that show court ceremonial and city views in detail
belonged to which of the following regional schools of art of painting?

(a) Kishangarh school

(b) Mewar school


(c) Marwar school

(d) Bundi school

EXPLANATION

Tamasha Paintings belonged to Mewar school that show court ceremonial and city views in detail.

Mewar school is one of the most significant schools of Indian painting of seventeenth and eighteenth
centuries. It was developed in Hindu principality of Mewar. The works of the Mewar School are
distinguished by simple vivid colour and straight poignant appeal. So, option (b) is correct.

Q.76) Consider the following statements:

1. It is a traditional painting art which shows a mix of classical and folk elements with
bias towards the latter.

2. The base material for the painting is treated cloth and colours used are derived from
natural sources only.

3. The unique feature of the painting is that it does not use pencil or charcoal and only
brush is used.

Identify the folk painting from the above given information:

(a) Patua art

(b) Kalighat painting

(c) Pattachitra

(d) Kalamkari painting

EXPLANATION

Pattachitra style of painting is one of the oldest and most popular art forms of Odisha.

1. The name Pattachitra has evolved from the Sanskrit words patta, meaning canvas, and
Chitra, meaning picture. Pattachitra is thus a painting done on canvas and is manifested by
rich colorful application, creative motifs, and designs, and portrayal of simple themes,
mostly mythological in depiction.

2. The Pattachitra style is a blend of both folk and classical elements but leans more towards
folk forms. The dress style has Mughal influences. All the poses have been confined to a few
well-defined postures

3. The base material for the painting is cloth mostly in form of a saree and natural colors
obtained from the gum of the Wood Apple (Kaitha) tree are used as a base for creating
different colors. Black colour is created with lamp soot while white colour is made from
powdered conch shells. Hartal stone is used to make yellow colour for Pattachitra art
paintings while the Gerua stone begets orange colour The leaves of Hyacinth bean plant are
used to make green

4. For painting the artists do not use any pencil or charcoal to sketch and only brush is used.
Root of the keya plant is usually used for making the common brush, while mouse hair is
used on the requirement of finer brushes, to be attached to wooden handles.

So, option (c) is correct.

ADDITIONAL INFORMATION

PATUA ART

 The art form of Bengal, Patua art dates back around a thousand years. It started out as
a village tradition by painters telling Mangal Kavyas or auspicious stories of Hindu Gods
and Goddesses.

 These paintings are done on pats or scrolls and for generations, the scroll painters or
patuas have been going to different villages to sing their stories. Most Patuas are
Muslims.

 Traditionally these were painted on cloth and told religious stories; today they are
painted with poster paints on sheets of paper sown together, usually to comment on
political and social issues.

 These Patuas mostly come from Medinipur region, Murshidabad, North and South 24
Parganas and Birbhum districts.

KALAMKARI PAINTING

 The name comes from kalam, i.e. a pen, which is used to paint these exquisite
paintings. The pen used is made of sharp pointed bamboo, used to regulate the flow of
colours.

 The base is cotton fabric while the colours used are vegetable dyes. the penis soaked in
a mixture of fermented jaggery and water; one by one these are applied and thereafter,
the vegetable dyes are applied.

 The main centers for this art are Srikalahasti and Machilipatnam in the State of Andhra
Pradesh.

 The images are drawn free hand and the inspiration comes from Hindu mythology.

 Textiles with handwork are also produced here. Kalamkari painting had its existence
even during Vijayanagara empire. It has received GI status.
KALIGHAT PAINTING

 Kalighat Paintings originated in 19th century Calcutta and were primitively used as
religious souvenirs for temple visitors.

 Traditionally hand-painted on cloth, these were art pieces produced by the patuas
(village artists). An elegant and bold art form, Kalighat art started out with religious
themes and depiction of gods, goddesses, and mythological characters but later with the
changing colonial atmosphere in Calcutta, the Kalighat artists started painting scenes
of daily lives which was their understanding of the contemporary change in people’s life.

Q.77) Consider the following statements:

1. Kashmir is famous for fine workmanship of Kashida embroidery which is done on


Cashmere shawls.

2. Phukari is an embroidery art which uses the technique of darning to create colourful
flowers like patterns on the cloth.

3. Kantha art is famous in West Bengal and Odisha.

Which of the above given statements is/are correct?

(a) 1 and 2 only

(b) 2 and 3 only

(c) 1 and 3 only

(d) 1,2 and 3

EXPLANATION

Kashmir is famous for the workmanship of Kashida embroidery which is done on Cashmere
shawl. Kashida, also known as Kasida, is one of the oldest forms of embroidery that
originated in Jammu and Kashmir. Kashida embroidery is created by using thick colored
threads as well as beads to create different patterns. Most kashida motifs that are used to
create this form of embroidery include images inspired by nature such as vines, birds, leaves,
and flowers and this is one of the defining aspects of this form of embroidery. So, statement
1 is correct.

Phulkari, which translates into ‘flower work’, is done in Punjab. The flower work is on
shawls, Kurtis, and chunris. The main characteristics of this embroidery are the use of darn
stitch on the wrong side of the cloth with colored silken thread. So, statement 2 is correct.

Kantha art practiced by women in rural Bengal and Odisha, Kantha was used to create
blankets (in Bangla, the word is used interchangeably for the embroidery and the blankets).
The patterns, crafted using a simple running stitch, are themed on daily life, floral and
animal motifs, and geometric shapes. So, statement 3 is correct.

Q.78) Consider the following:

1. Archeological site Nalanda Mahavira

2. Rani ki Vav

3. Rock shelters of Bhimbetka

4. Brihadeshwara temple

Arrange the above sites in the ascending order of the year in which they were listed as a
UNESCO heritage site:

(a) 4-3-2-1

(b) 3-4-2-1

(c) 1-2-3-4

(d) 1-2-4-3

EXPLANATION

1. Archeological site Nalanda Mahavira – It got listed under UNESCO World Heritage in
the year 2009. The Nalanda Mahavihara site is in the State of Bihar, in north-eastern
India. It comprises the archaeological remains of a monastic and scholastic institution
dating from the 3rd century BCE to the 13th century CE. It includes stupas, shrines,
viharas (residential and educational buildings) and important art works in stucco, stone
and metal. Nalanda stands out as the most ancient university of the Indian
Subcontinent. It engaged in the organized transmission of knowledge over an
uninterrupted period of 800 years. The historical development of the site testifies to the
development of Buddhism into a religion and the flourishing of monastic and
educational traditions.

2. Rani ki Vav - It got listed under UNESCO World Heritage in the year 2014. Rani-ki-Vav,
on the banks of the Saraswati River, was initially built as a memorial to a king in the
11th century AD. Stepwells are a distinctive form of subterranean water resource and
storage systems on the Indian subcontinent, and have been constructed since the 3rd
millennium BC.

3. Rock shelters of Bhimbetka - It got listed under UNESCO World Heritage in the year
2003. The Rock Shelters of Bhimbetka are in the foothills of the Vindhyan Mountains on
the southern edge of the central Indian plateau. Within massive sandstone outcrops,
above comparatively dense forest, are five clusters of natural rock shelters, displaying
paintings that appear to date from the Mesolithic Period right through to the historical
period. The cultural traditions of the inhabitants of the twenty-one villages adjacent to
the site bear a strong resemblance to those represented in the rock paintings.

4. Brihadeshwara temple - It got listed under UNESCO World Heritage in the year 1987.
The Great Living Chola Temples were built by kings of the Chola Empire, which
stretched over all of south India and the neighbouring islands. The site includes three
great 11th- and 12th-century Temples: the Brihadisvara Temple at Thanjavur, the
Brihadisvara Temple at Gangaikondacholisvaram and the Airavatesvara Temple at
Darasuram. The Temple of Gangaikondacholisvaram, built by Rajendra I, was
completed in 1035. Its 53-m vimana (sanctum tower) has recessed corners and a
graceful upward curving movement, contrasting with the straight and severe tower at
Thanjavur.

The ascending order will be Brihadeshwara temple - Rock shelters of Bhimbetka - Archeological
site Nalanda Mahavira - Rani ki Vav. So, option (a) is correct.

Q.79) Consider the following:

1. Gandharva veda

2. Aitareya Aranyaka

3. Mudrarakshasa

4. Natyashastra

Which of the above given texts is/are related to the Indian music?

(a) 1,2 and 3 only

(b) 1 and 2 only

(c) 1,2 and 4 only

(d) 3 and 4 only

EXPLANATION

Gandharva Veda, Aitareya Aranyaka, Bharata's Natyashastra are related to Indian Music.
Mudrarakshasa is a historical play in Sanskrit by Vishakhadatta narrates the ascent of king
Chandragupta Maurya to power in Northern India.
So, option (c) is correct.
Q.80) With reference to difference between Hindustani music and Carnatic music,
consider the following statements:

1. Hindustani music has influence from Arab, persian and Afghan music but the Carnatic
music is totally indigenous.

2. While Hindustani music places more emphasis on vocals, Carnatic music places equal
importance to both instruments and vocals.

3. While the former adheres to time, the latter does not adhere to time.

Which of the above given statements is/are correct?

(a) 1 only

(b) 1 and 3 only

(c) 1 and 2 only

(d) 2 and 3 only

EXPLANATION

Historical roots of both Hindustani and Classical music type belongs to Bharata’s Natyashastra,
they diverged in the 41th century. Hindustani music has influence from Arab, Persian, and
Afghan music but the Carnatic music is indigenous. So, statement 1 is correct.
While Carnatic music places more emphasis on vocals, Hindustani music places equal
importance to both instruments and vocals.So, statement 2 is not correct.

Hindustani music adheres to time where Carnatic music does not adhere to time. In Hindustani
classical music, each Raag is rendered only at a specific time. So, statement 3 is correct.

Q.81) With reference to the elements of Indian dance, which of the following statements
is/are correct?

1. Nritta refers to basic dance steps and is devoid of any expressions.

2. Natya is the dramatic representation and seeks to elaborate the story through the dance.

3. Nritya displays the sentiments and emotions which are evoked through the dance.

Select the correct answer using the code given below:

(a) 1 and 2 only

(b) 2 only
(c) 1 and 3 only

(d) 1,2 and 3

EXPLANATION

Nritta refers to the basic dance steps, performed rhythmically but devoid of any expression or
mood. So, statement 1 is correct.

Natya means dramatic representations and refers to the story that is elaborated through the
dance recital. So, statement 2 is correct.

Nritya refers to the sentiment and the emotions evoked through dance. It includes the mime and
the different methods of expression including mudras in the dance. So, statement 3 is correct.

Q.82) ‘Atavakul’ or ‘Atavus’, which is the collection of forty basic dance movements, is
related to which of the following classical dances?

(a) Mohiniattam

(b) Kathakali

(c) Kuchipudi

(d) Bharatnatyam

EXPLANATION

‘Atavakul’ or ‘Atavus’, is the collection of 40 basic dance movements that are related to the
feature of Mohiniattam. So, option (a) is correct.

ADDITIONAL INFORMATION

Mohiniattam

 Mohiniattam is a solo dance performance by women.


 It was developed by Vadivelu in the 19th century and gained prominence under the
rulers of Travancore.
 It combines the grace and elegance of Bharatnatyam and Kathakali.
 There is an absence of thumping of footsteps and footwork is gentle.
 The element of air is symbolized through Mohiniattam's performance.
 Musical instruments used are Cymbals, Veena, drums, flute.
 The dance is accompanied by music and songs.
Q.83) Consider the following pairs:

Folk dance State/region

1. Kalbelia - Rajasthan

2. Jawara - Odisha

3. Thang Ta. - Nagaland

4. Padayani - Kerala

Which of the above given pairs is/are correctly matched?

(a) 1 and 2 only

(b) 2 and 3 only

(c) 1 and 4 only

(d) 3 and 4 only

EXPLANATION

1. Kalbelia - It is a sensuous folk dance performed by the women of the Kalbelia


community of Rajasthan.
2. Jawara – Jawara is the harvest dance that is famous in the Bundelkhand region of
Madhya Pradesh.
3. Thang Ta – It is a martial dance form of Manipur. Thang means sword and Ta means
spear.
4. Padayani – It is a dance performed in the Bhagavati temples of Southern Kerala.
Padayani means infantry and it is a very rich and colorful affair.
Therefore 1 and 4 are correctly matched. So, option (c) is correct.

Q.84) Who among the following kings were the contemporaries of Gautama Buddha?

1. Bimbisara

2. Ajatsatru

3. Chandragupta Maurya

4. Ashoka

Select the correct answer using the code given below:

(a) 1 and 2 only


(b) 2 only

(c) 1 and 3 only

(d) 2 and 4 only

EXPLANATION

Ajatsatru and Bimbisara were the contemporaries of both Gautama Buddha and Vardhamana
Mahavira. Chandragupta Maurya and Ashoka are not the contemporaries of Gautama Buddha.
So, option (a) is correct.

Q.85) With reference to Mauryan administration, consider the following statements:

1. The King was assisted by a council of ministers called Mantriparsihad and it consisted of
Purohita, Mahamantri, Senapati and Yuvaraja.

2. The land revenue was fixed at one sixth of the total produce.

3. There is no evidence of the taking of census during the Mauryan period.

Which of the above given statements is/are correct?

(a) 1 and 2 only

(b) 2 and 3 only

(c) 1 and 3 only

(d) 1,2 and 3

EXPLANATION

The Mauryan administration is highly centralized. The king was assisted by the Council of
Ministers called Mantriparsihad. The Mantriparsihad consists of Mahamantri(minister),
Purohita(high priest), Senapati(Commander-in-Chief), and Yuvaraja(crown prince). So,
statement 1 is correct.
The revenues came from the land, irrigation, customs, shop tax, ferry tax, forests, mines and
pastures, license fees from craftsmen, and fines collected in the law courts. The land revenue
was normally fixed as one-sixth of the produce. So, statement 2 is correct.
Taking Census was regular during the Mauryan period. The village officials collect information
about the people like caste and occupation. It also includes information about Animals in the
houses. So, statement 3 is not correct.
Q.86) With respect to Sangam society in ancient India, consider the following pairs:

Social Division Occupation

1. Kurinji - Hunting and Honey

Collection

2. Murudam - Cattle rearing

3. Neydal - Fishing

4. Palai - Agriculture

Which of the above given pairs is/are correctly matched?

(a) 1 and 2 only

(b) 1 and 3 only

(c) 3 and 4 only

(d) 2 and 4 only

EXPLANATION

Kurinji - The landscape is a mountainous region with a special flower named kurinji that
blossoms once in twelve years. Monkeys, elephants, and bulls are the animals of the location
and are filled with bamboos, jack fruit trees, and ‘venkai’ trees. The occupants are tribal
people who hunt and gather honey.
Marudam - It is agricultural pasture land with mango trees and ponds brimming with water.
The freshwater fish and water buffaloes are in abundance here. Agriculture is the main
occupation of the people residing here.
Neydal - The place is filled with ‘punnai’ trees, crocodiles, and sharks. The inhabitants are
the fisher who goes deep into the sea to catch fish.
Palai – It is a desert region and the main occupation engaged here is robbery.
Therefore 1 and 3 are correctly matched. So, option (b) is correct.

Q.87) Consider the following statements with regard to Gupta king Samdragupta:

1. The Allahabad pillar inscription is an important source of the reign of Samudragupta.

2. He was a great patron of poets and scholars like Harisena and vasubandhu.
3. He assumed the title of Sakari after destroying sakas and also called himself Vikramaditya.

Which of the above given statements is/are not correct?

(a) 1 only

(b) 2 and 3 only

(c) 3 only

(d) None

EXPLANATION

Information about the Gupta period is available from both archaeological and literary sources.
The most important archaeological source of the Gupta period is the Allahabad Pillar inscription.
It gives the details of Samudragupta's military expeditions. So, statement 1 is correct.
Samudragupta was a patron of many poets and scholars, which include Harisena and
Vasubandhu. So, statement 2 is correct.
Chandragupta II adopted the title Vikramaditya which has been first used by the Ujjain ruler in
58 – 57 BC as a mark of victory over the Shaka Kshatrapas of western India. So, statement 3 is
not correct.

Q.88) ‘It is a ritualistic theatre of the Garhwal region in the state of Uttarakhand and is
dedicated to Bhumiyal devta, the local deity of the region. People of Bhandari caste
wear a sacred mask which symbolises Narasimha. The art form is listed in the
UNESCO representative list of the intangible cultural heritage of humanity.’

Identify the theatre art from from the above given description:

(a) Ankia Nat

(b) Bhavai

(c) Ramman

(d) Tamasha

EXPLANATION

1. The Ramman is a multiform cultural event that reflects the environmental, spiritual, and
cultural concept of the community, recounting its founding myths and strengthening its
sense of self-worth.
2. Ramman is a form of traditional ritual theatre performed every year in the courtyard of the
temple of Bhumiyal Devta. It is situated in Saloor Dungra Village in Painkhanda valley of
Chamoli district, Uttarakhand, India.
3. The village deity of Saloor Dungra is Bhumichetrapa, where he is better known as Bhumiyal
Devta. It is in this temple where every year Ramman festival is organized by the local
inhabitants.
4. On the auspicious day of Sankranti (Baisakhi) in the month of Baisakh, Bhumiyal Devta
comes out in a procession from his place of residence (which is one house in the village) to
the central temple of the village accompanied by beating of drums and mask dances.
5. The sacred mask denotes Narasimha.
6. It is listed in the UNESCO representative list of the intangible cultural heritage of humanity
in 2009.
7. People of the Bhandari caste wear sacred masks symbolizing Narasimha, the half-
man, half-lion avatar of Lord Vishnu.
So, option (c) is correct.

Q.89) Consider the following statements with respect to Indian Puppetry:

1. The oldest reference to Indian puppetry is found in Tamil classic named


Silappadikaram.

2. Ravan Chhaya is a type of String puppetry.

3. Absence of legs is a unique feature of Kathputli.

Which of the above given statements is/are correct?

(a) 1 and 2 only

(b) 1 and 3 only

(c) 2 and 3 only

(d) 1,2 and 3

EXPLANATION

The written reference of Indian Puppetry is found in the Tamil classic Silappadikaram, which is written
in the 1st and 2nd century BC, and the Puppetry is also mentioned in Mahabharata. So, statement 1
is correct.
Ravan Chhaya is a type of Shadow Puppetry. It is popular in entertainment in the Odisha region. The
puppets are made of deerskin and depict bold and dramatic postures. So, statement 2 is not correct.
Kathputli is a traditional string puppet of Rajasthan. Kath meaning wood and putting meaning doll. A
unique feature of the puppets in Kathputli is the absence of legs. The strings are attached to the finger
of the puppeteer. So, statement 3 is correct.

Q.90) Consider the following statements with regard to imperial Cholas:

1. Rajaraja I was the founder of imperial cholas with its capital at Tanjore.

2. Rajaraja I completed the construction of Brihadeshwara temple in 1010 A.D and also
helped in the construction of Buddhist monastery at Nagapattinam.

3. Rajendra I founded the city of Gangaikondacholapuram and constructed the famous


Rajesvaram temple in the city.

Which of the above given statements is/are correct?

(a) 1 and 2 only

(b) 2 and 3 only

(c) 1 and 3 only

(d) 1,2 and 3

EXPLANATION

The founder of the Imperial Chola line was Vijayalaya. He captured Tanjore from Muttaraiyars in
815 A.D and established Tanjore as the Capital. So, statement 1 is not correct.
Rajaraja Chola I constructed the Brihadisvara Temple in the 10th century A.D., designed by the
famous architect Sama Varma. He also helped in the construction of a Buddhist monastery at
Nagapattinam. So, statement 2 is correct.
Chola's army crossed the Ganges by defeating several rulers on their way. Rajendra defeated
Mahipala of Bengal. To commemorate this successful north Indian campaign Rajendra founded
the city of Gangaikondacholapuram and constructed the famous Rajesvaram temple in that city.
He also excavated a large irrigation tank called Cholagangam on the western side of the city. So,
statement 3 is correct.

Q.91) With reference to early medieval India, consider the following statements:

1. The battle of Tarain and battle of Chandawar contributed to the establishment of


Turkish rule in India.

2. The continued reliance of Hindu rulers in elephants and lack of unity were the major
reasons for the defeat of Hindu kingdoms.
3. Muhammad Ghori appointed his general Qutb-ud- din Aibak to expand the Turkish
rule in India.

Which of the above given statements is/are correct?

(a) 1 only

(b) 2 and 3 only

(c) 1 and 3 only

(d) 1,2 and 3

EXPLANATION

Battle of Tarain was fought between Prithiviraj Chauhan of the Chauhan dynasty of Ajmer and
Muhammad Ghori. The Battle of Chandawar (1193 or 1194) was fought between Mohammad
Ghori and Jaichand of Kannauj of the Gahadavala dynasty. The Battles of Tarain and Chandawar
contributed the most to the establishment of Turkish rule in India. So, statement 1 is correct.
The main reason for the defeat of the Hindu lack of better technologies such as catapults, artillery,
and gun powder which were used by the enemies. The Hindu rulers mainly relied on elephants were
enemies used horses and there is a lack of unity among the rulers. So, statement 2 is correct.
Muhammad Ghori appointed his general Qutbuddin Aibak, a Turkish slave in 1206 to look after
the territories conquered by him, Aibak played an important role in the expansion of the Turkish
Sultanate in India after the battle of Tarain. So, Statement 3 is correct.

Q.92) The archive of which of the following king’s period is called ‘Nilopitu’?

(a) Kanishka

(b) Chandragupta II

(c) Narasimhavarman I

(d) Harshavardhan

EXPLANATION

The maintenance of public records was the salient feature of Harsha’s administration. The
archive of the Harshavardhan period was known as nilopitu and it was under the control of
special officers. Both good and bad events that happened during his time had been recorded. So,
option (d) is correct.

Q.93) In the context of Delhi Sultanate rule in India, who among the following created a
new class of ruling elite of forty powerful military leaders?
(a) Iltutmish

(b) Alauddin Khalji

(c) Muhammad bin Tughlaq

(d) Balban

EXPLANATION

Illtutmish created a new class of ruling elite of forty powerful military leaders. So, option (a) is
correct.

Q.94) With respect to the administration of Alauddin Khalji, which of the following
statements is/are correct?

1. Munhiyans were secret agents who were appointed to look after the functioning of
markets setup by Khalji.

2. Dagh was the descriptive list of soldiers recruited under the army.

3. Huliya as a system of branding the horses.

Select the correct answer using the code given below:

(a) 1 and 2 only

(b) 1 only

(c) 2 and 3 only

(d) 1,2 and 3

EXPLANATION

Market reforms were done by Sultan Alauddin Khalji. He set up Munhiyans who are the secret spies
reporting to the Alauddin Khalji regarding the functioning of the Market. An officer (Shehna) was in
charge of the market to see that no one violates the royal orders. Barids (intelligence officers ) were
also appointed. So, statement 1 is correct.
Dagh was the system of branding horses and animals. So, statement 2 is not correct.

Huliya was the descriptive list of soldiers recruited under the army. So, statement 3 is not correct.

Q.95) Which of the following Bhakti saints belonged to Varkari panth?

1. Jnaneshwar
2. Ramananda

3. Namdev

4. Eknath

Select the correct answer using the code given below:

(a) 1,2 and 3 only

(b) 2,3 and 4 only

(c) 1,3 and 4 only

(d) 1,2,3 and 4

EXPLANATION

Jnaneshwar, Namdenv, and Eknath are the Bhakti saints who belonged to Varkari panth.
 Varkari panth is are the cults belonging to the lower caste of Maharashtra
 Jnanesvara is the first of a long line of writers and poets whose works are central to the
doctrine and practice of the Vitthalas, and he is often described as the founder of the
cult.
 Namdev (c. 1270-1350) helped to spread the bhakti movement in Punjab, where he
lived for twenty years. Some of his poems are found in Sikh texts and he is sometimes
considered a saint, holy man, by Sikhs and Sufis.
 Eknath (1533-99) spent his life in Maharastra. Besides writing hymns he translated the
Ramayana into Marathi for the first time and edited a reliable version of Jnanesvara’s
commentary on the Bhagavad Gita.
So, option (c) is correct.

Q.96) Consider the following statements with respect to Lingayatism sect:

1. It is a distinct Shaivite tradition which believes in monotheism through worship centred


on Lord Shiva in the form of Linga.

2. It rejects the authority of the vedas and the caste system.

Which of the above given statements is/are not correct?

(a) 1 only

(b) 2 only

(c) Both 1 and 2

(d) Neither 1 nor 2


EXPLANATION

The term Lingayat denotes a person who wears a personal linga, an iconic form of god Shiva, on the
body which is received during the initiation ceremony. Lingayats are the followers of the 12th-century
social reformer-philosopher poet, Basaveshwara. Basaveshwara was against the caste system and
Vedic rituals. The Lingayats are strict monotheists. They enjoin the worship of only one God, namely,
Linga (Shiva).The word ‘Linga’ does not mean Linga established in temples, but universal
consciousness qualified by the universal energy (Shakti).Lingayats had been classified as a Hindu
subcaste called “Veerashaiva Lingayats” and they are considered to be distinct Shaivites.So, both the
statements are correct. So, option (d) is correct.

Q.97) Which of the following Bodhisattvas is described as ‘Bodhisattvas of Compassion’?

(a) Manjusri

(b) Avalokitesvara

(c) Vajrapani

(d) Maitreya

EXPLANATION

Bodhisattvas are enlightened beings who are destined to become buddhas, but postpone that final state
in order to help humanity.
 Avalokiteshvara is described as Bodhisattvas of Compassion
 The name Avalokiteshvara means “Lord who looks down with compassion.”
So, option (b) is correct.

Q.98) Which of the following is/are sub-sects of Digambaras sect under Jainism?

1. Terapanthi

2. Mula singh

3. Sthanakvasi

4. Murtipujaka

Select the correct answer using the code given below:

(a) 1 and 2 only

(b) 2,3 and 4 only

(c) 1 and 3 only


(d) 2 and 4 only1

EXPLANATION

Jainism is dived into Two sects namely Digamabaras and Svetambaras.


The sub-sects of Digambaras include
 Mula singh and Terapanthi
 Tarapanthi and Bispanthi
The sub-sects of Svetambaras includes
 Sthanakavasi and Murtipujaka
Therefore Sthanakavasi and Murtipujaka are the sub-sects of Svetambaras. So, option (a) is
correct.

Q.99) In the context of Indian literature, consider the following statements:

1. The rules of Sanskrit grammar are elaborately defined in Panini's Ashtadhyayi.

2. Upanishads are the last part of vedas and provide a path to human salvation.

Which of the above given statements is/are

(a) 1 only

(b) 2 only

(c) Both 1 and 2

(d) Neither 1 nor 2

EXPLANATION

Sanskrit literature was bound by the rules of grammar that have been explained in Panini’s
Ashtadhyayi, a treatise on the rigid rules which bind the Sanskrit language. So, statement 1 is
correct.
Upanishads are generally the last part of the Vedas, they are also known as Vedanta. The Upanishads
are said to have the ‘truth’ about human life and show the way towards human salvation or Moksha.
So, statement 2 is correct.

Q.100) ‘It is a political drama which narrates the ascent of king Chandragupta Maurya to
power in India. It was written by Vishakadutta.

Identify the sanskrit drama described in the above passage:


(a) Mricchakatika

(b) Mudra Rakshasa

(c) Ratnavali

(d) Malavikagnimitra

EXPLANATION

The Mudrarakshasa written by Visakadatta is a drama in Sanskrit. Although written during the
Gupta period, it describes how Chandragupta with the assistance of Kautilya overthrew the
Nandas. It also gives a picture of the socio-economic condition under the Mauryas. So, option (b)
is correct.

ADDITIONAL INFORMATION

Mricchakatika

 The Mricchakatika (the clay-cart) by Sudraka (248 A.D.) presents a remarkable social
drama with touches of grim reality. The characters are drawn from all stratas of society,
which include thieves and gamblers, rogues and idlers, courtesans and their associates,
police constables, mendicants and politicians

Ratnavali

 Ratnavali is a Sanskrit drama about a beautiful princess named Ratnavali, and a great
king named Udayana. It is attributed to the Indian emperor Harsha (606–648). It is a
Natika in four acts. One of the first textual references to the celebration of Holi, the
festival of Colours have been found in this text.

You might also like